Vous êtes sur la page 1sur 324

M.

Sc (Mathematics) Second Year


Paper-VII Analysis - III

Welcome

Dear Students,

We welcome you as a student of the

Second year M.Sc degree course.

This paper deals with the subject

ANALYSIS III. The learning

material for this paper will be

supplemented by contact lectures.

In this book the first five units deal

with Real analysis and the last five

units deal with Measure Theory.

Learning through the Distance

Education mode, as you are all

aware, involves self learning and self


assessment and in this regard you

are expected to put in disciplined and

dedicated effort.

As our part, we assure of our

guidance and support.

With best wishes,


SYLLABUS

M.Sc., Second Year

Paper VII Analysis III

Unit 1: Definitions and existence of the

Integral, Properties of the Integral,

Integration and differentiation,

Integration of vector valued

functions, Rectifiable curves

(Chapter 6 : Sections 6,1 to 6.23)

Unit 2: Discussion of the main problem,

uniform convergence, Uniform

convergence and continuity,

uniform convergence and

integration, uniform convergence

and Differentiation, Equi continuous

families of functions, The Stone

Weierstrass Theorem (Chapter 7:

Sections 7.1 to 7.33)

Unit 3: Power series, The exponential and


Logarithmic functions, The
trigonometric Functions, The

algebraic completeness of the

Complex field, Fourier series, The

Gamma function (Chapter 8:

Sections 8.1 to 8.22)

Unit 4: Linear Transformations

Differentiation The contraction

principle - The inverse function

theorem (Chapter 9 relevant

sections)

Unit 5: The implicit function theorem The

rank theorem Determinants

Derivatives of higher order

Differentiation of integrals (Chapter

9 relevant sections)

Unit 6: Lebesgue outer measure

Measurable sets Regularity.

Unit 7: Measurable functions Borel and

Lebesgue measurability.
Unit 8: Integration of non-negative

functions The general integral

Integration of series

Unit 9: Riemann and Lebesgue integrals

The four derivatives Continuous

nondifferentiable functions.

Unit 10: Functions of bounded variations

Lebesgue differentiation theorem

Differentiation and integration

The Lebesgue set

Text Books:

1. Principles of Mathematical

Analysis by Walter Rudin , Third

Edition, McGraw Hill ,

International Student Edition,

1976 Chapters 6,7,8,9.


2. Measure Theory and Integration

by G. de Barra, Willey Eastern

Ltd 2 edition 1991

Chapters 2,3 and 4


SCHEME OF LESSONS

ANALYSIS III

S.
TITLE
No

Unit 1

1.1: Definitions and existence of the


1
Integral

2 1.2 : Properties of the Integral.

3 1.3 : Integration and differentiation

1.4 : Integration of vector valued


4
functions

5 1.5 : Rectifiable curves

Unit 2

6 2.1 : Discussion of the main problem

7 2.2: Uniform convergence


8 2.3: Uniform convergence and continuity

2.4: uniform convergence and


9
integration

2.5: uniform convergence and


10
Differentiation

11 2.6: Equi continuous families of functions

12 2.7: The Stone Weierstrass Theorem

Unit 3

13 3.1: Power series

3.2: The exponential and Logarithmic


14
functions

15 3.3: The trigonometric Functions

3.4: The algebraic completeness of the


16
Complex field

17 3.5: Fourier series


18 3.6: The Gamma function

Unit 4

19 4.1: Linear Transformations

20 4.2: Differentiation

21 4.3: The contraction principle.

22 4.4: The inverse function theorem

Unit 5

23 5.1: The implicit function theorem

24 5.2: The rank theorem

25 5.3: Determinants

26 5.4: Derivatives of Higher Order

27 5.5: Differentiation of integrals


UNIT-1

Unit Structure

Section 1.1 : Definitions and

existence of the Integral

Section 1.2: Properties of the

Integral.

Section 1.3 : Integration and

differentiation

Section 1.4: Integration of

vector valued functions

Section 1.5 : Rectifiable curves

Introduction

A satisfactory discussion of the main

concepts of analysis must be based

on an accurately defined number


concept. In this unit we discuss the

concept of Riemann-Stieltjes integral

and its properties, some theorems of

Integration of vector valued functions

and Rectifiable curves.

SECTION-1.1 DEFINITIONS &


EXISTENCE OF THE INTEGRAL

Definition:

Let [a,b] be a given interval, by a

partition P of [a,b] we mean a finite

set of points x 0 , x 1 , x 2 ,...,x n where

a=x 0 x 1 x 2 ...x n = b

We write x i = x i x i1 for i = 1,2,..

.,n.
Suppose f is a bounded real function

defined on [a,b] corresponding to

each partition P of [a,b] we put

Mi = supf(x), (xi-1 x xi), mi = inf f(x), (xi-1 x xi),


n n

U(P,f) = M x , L(P,f) = m x
i i i i
i=1 i=1
b
_

and finally put fdx = inf U(P, f) ------------------(1)


a
b

fdx = sup L(P,f) ------------------(2)



a

where the inf and sup are taken over

all partitions P of [a,b]. The left

members of (1) and (2) are called

Upper and Lower Riemann

integrals.If the upper and lower

integrals are equal, we say that f


is Riemann integrable on [a,b], we

write f and we denote the

common value of (1) & (2)


b b

fdx or f(x)dx .This is the Riemann


a a

integral of f over [a,b].

Since f is bounded, there exist m and

M such that m f(x) M (axb).

Hence, for every P, m(b a) L(P,f)

U(P,f) M(b a), so that the

numbers L(P,f) and U(P,f) form a

bounded set. That is, the upper and

lower integrals are defined for every

bounded function f.

Definition:

Riemann-Stielties integral
Let be monotonically increasing

function on [a,b]. Corresponding to

each partition P of [a,b] we write

i = (x i ) (x i1 ) for i = l,2,...,n.

Suppose f is a bounded real function

on [a,b] we put

n n

U(P,f,) = M , L(P,f,) = m and we define


i i i i
i=1 i=1
b
_

fd = inf U(P,f,) -------------------(3)


a
b

fd = sup L(P,f,) -------------------(4)



a

where inf and sup are taken over

all partitions P of [a,b]. The left

members of (3) and (4) are called the

upper and lower Riemann-Stieltjes


integral of f with respect to , over

[a,b] and we write f ().


If the left members of (3) and (4)

are equal we say that f is Riemann-

Stieltjes integrable with respect to ,

over [a,b].

Remark:

By taking (x) = x, the Riemann

integral becomes a special case of

Riemann-Stieltjes integral.

Definition:

We say that the partition P* is a

refinement of P if P* P.

Given two partitions P 1 and P 2 we say

that P* is there common refinement

if P* = P 1 P 2 .
Theorem 1.1.1:

If P* is a_refinement of P then

L(P,f,) L(P*,f,) and U(P*,f,)

U(P,f,).

Proof:

Let P = (a= x 0 , x 1 , x 2 ,..x n = b} be a

partition of [a,b].

Let P* be a refinement of P. Then

P*P.

Suppose first that P* contains just

one point more that P.

Let this extra point be x* and

x i1 x* x i where x i1 and x i are two

consecutive points of P.
Put w1 = inf(f(x)), xi-1 x x*

w2 = inf (f(x)), x* x xi

and mi = inf(f(x)), xi-1 x xi

Now L(P*,f,) L(P,f,)


[ ] [
=w1 (x*) (xi-1) + w2 (xi) (xi-1) ]
[ ] [
=w1 (x*) (xi-1) + w2 (xi) (x*) ]
[
-mi (xi) (xi-1) + (x*) (x*) ]
[ ] [
=(w1 mi) (x*) (xi-1) + (w2 mi) (xi) (x*) ]
0.
(i.e)L(P*,f,) L(P,f,) 0.
L(P*,f,) L(P,f,)
L(P*,f,) L(P,f,)

If P* contains k points more than P ,

we repeat this reasoning k times and

arrive at a result L(P*,f ) L(P,f,).

Similarly, we can prove that

U(P*,f,) U(P,f,).

Theorem 1.1.2:
b
b _

fd fd
a
a

Proof:

Let P* be the common refinement of


two partitions P 1 and P 2 .
Then L(P 1 ,f,) L(P*,f,)

U(P*,f,) U(P 2 ,f,).


Then L(P1,f,) L(P,f,) U(P*,f,) U(P2,f,)

Hence L(P1, f,) U(P2, f,) ------------(1)

If P 2 is fixed and supremum is taken

over all P 1 , (1) gives sup L(P 1 ,f,)

U(P 2 ,f,) .

sup L(P1, f,) U(P2, f,)


b

(i.e)fd U(P2, f,) -----------------(2)



a

By taking inf over all P 2 in (2), we get


b

fd infU(P , f,) 2

b b

fd fd

a a

Theorem 1.1.3:
( Necessary and Sufficient condition

for Riemann-Stieltjes integrability)


f () on [a,b] if and only if for

every > 0, there exist a partition P

such that U(P,f,) L(P,f,) < .

Proof:

Necessary Condition:

Let f ().

Then, by definition
b
b _

fd = fd ----------------(1)
a
a

Since fd is the supremum of



a

L(P,f,) over all partitions P , there

exist a partition P1 such that


b

fd < L(P1, f,) +



2

a
b
_

Also , since fd is the infimum of


a

U(P,f,) over all partitions P ,there

exist a partition P2 such that


b
_

U(P 2 ,f,) < fd +



2
a

If P = P 1 P 2 , then P is the common

refinement of P 1 and P 2

Then, by definition of refinement, we

have



fd < L(P,f,) + 2 ------------------(2)

a
b
_



and U(P,f,) < fd + 2 ------------ (3)
a

Adding (2) and (3), we get


b
b _

fd + U(P,f,) < L(P,f,) + fd +


a
a

By (1), we have
U(P,f,) < L(P,f,) +

(i.e)U(P,f,)-L(P,f,)<

Sufficient Condition

Let U(P,f,) L(P,f,) < .

For every partition P we have


b
b _

L(P,f,) fd fd U(P,f,)
a
a

b b
_ _

fd fd U(P,f,) L(P,f,) <


a a
b
b _

(i.e)fd fd <
a
a
This is true for > 0, we have
b
b _

fd = fd.
a
a

Hence f () on [a,b].

Theorem 1.1.4:
a. If U(P,f,) L(P,f,) < holds

for some P and some then the

inequality holds for every

refinement P.

b. If If U(P,f,) L(P,f,) < holds

for P = { x 0 , x 1 x 2 ,..., x n } and

if s i ,t i are arbitrary points in

[x i1 ,x i ] then
n

|f(s ) f(t )| <


i i i
i=1
c. If f () and t i [x i1 ,x i ] then

| |
n b

f(t ) fd
i i <
i=1 a

Proof:

a. Let U(P,f,) L(P,f,) < for

some partition P and some > 0.

Let P* be the refinement of P.

Then U(P*,f,) U(P,f,) and

L(P,f,) L(P*,f,) .

We have U(P*,f,) L(P*,f,)

U(P,f,) L(P,f,) < .

U(P*,f,) L(P*,f,) < .

Hence the result holds good for


every refinement of P.

b. Let s i ,t i he two arbitrary points

in [x i1 ,x i ].
|
Then f(si) f(ti) Mi mi |
n n

|f(s ) f(t )| (M m )
i i i i i i
i=1 i=1

n n

(M ) (m )
i i i i
i=1 i=1

U(P,f,) L(P,f,) <


n

(i.e) |f(si) f(ti)| i < .


i=1

Let ti [xi-1, xi], i=1,2,....,n

Then Mi f(ti) mi i=1,2,....,n


n n n

m f(t ) M
i i i i i i
i=1 i=1 i=1
n

L(P,f,) f(t ) U(P,f,)


i i ---------(1)
i=1
b
b _
c.
Also L(P,f,) fd fd U(P,f,).
a
a
b

L(P,f,) fd U(P,f,) ------------(2)


a

From (1) and (2), we get

| |
n b

f(ti)i fd U(P,f,) L(P,f,) <


i=1 a
Theorem 1.1.5:

If f is continuous on [a,b] then f

() on [a,b].

Proof:

Let > 0 be given.

Choose > 0 so that ((b) (a))

< .

Since f is continuous on [a,b] and

[a,b] is compact, f is uniformly

continuous.

There exists a > 0 such that x,t

[a,b] and

|x-t| < |f(x) f(t)| < -------(1)

If P is any partition of [a,b] such that

x i < , i, then (1) implies


Mi mi <
n n

U(P,f,) L(P,f,) = M m
i i i i
i=1 i=1
n

= (M -m )
i i i
i=1

< i
i=1

= (1 + 2 + ... + n)

(
= (x1) (x0) + (x2) (x1) + ... + (xn) (xn-1) )
(
= (xn) (x0) )
= ((b) (a))

<
U(P,f,) L(P,f,) <

By theorem 1.1.3, f () on [a,b].

Theorem 1.1.6:

If f is monotonic on [a,b] and if is

continuous on [a,b] then f ().

Proof:

Let > 0 be given.

Double click this page to view clearly


Since is continuous on [a,b], for

any positive integer n, choose a

partition such that


(b) (a)
i = n , i=1,2,....,n

By hypothesis f is monotonic on

[a,b].

Suppose f is monotonically increasing

(the proof is analogous in the other

case). Then Mi = f(x i ) and mi=

f(x i1 ).
n n

U(P,f,) L(P,f,) = M m
i i i i
i=1 i=1
n

= (M m ) i i i
i=1
n

(f(x ) f(x ))
(b) (a)
= i i-1 n
i=1

(b) (a)
= n f (xn) f(x0)
(b) (a)
= n f (b) f(a)
<, if n is taken large enough.
By theorem 1.1.3, f () on [a,b].

Theorem 1.1.7:

Suppose f is bounded on [a,b] , f


has only finitely many points of

discontinuity on [a,b], and is

continuous at every point at which f

is discontinuous. Then f () .

Proof:

Let > 0 be given.

Put M = sup|f(x)|, let E be the set


of points at which f is discontinuous.

Since E is finite and is continuous

at every point of E, we can cover

E by finitely many disjoint intervals

[u j ,v j ] [a,b] such that the sum of


the corresponding differences (v j )

(u j ) < .

Further more , we can place these

intervals in such a way that every

point of E (a,b) lies in the interior

of some [u j ,v j ].

Remove the segments (u j ,v j ) from

[a,b]. The remaining set K is


compact. Hence f is uniformly

continuous on K, and there exists >

0 such that |f(s) f(t)| < if sK,

tK, |s t| < .

Now form a partition P = {x 0 , x 1 ,

x 2 ,..x n } of [a,b] as follows. Each u j

occurs in P. Each v j occurs in P. No

point of any segment (u j v j ) occurs in

P. If x i1 is not one of the u j , then x i

< .
Note that M i m i 2M for every i,

and that M i m i < unless x i1 is one

of the u j .

Hence U(P,f,) L(P,f,) [(b)

(a)] + 2M.

Since > 0 is arbitrary, by theorem

1.1.6, f ().

Theorem 1.1.8:

Suppose f () on [a,b], m f

M, is continuous on [m, M] and

h(x) = f(x)) on [a,b]. Then he h

() on [a,b].

Proof:

Choose > 0.

Since is continuous on [m, M] and

[m, M] is compact, is uniformly

continuous on [m, M]
there exist > 0 such that <

and s,t [m, M] and

|s 1| < |(s) (t)| < .

Since f () on [a,b], there is a

partition P of [a,b] such that

2
U(P,f,) L(P,f,) < . ------------------(1)
Let Mi = sup f(x), ,xi-1 x xi,

mi = inf f(x), xi-1 x xi,


Mi * = sup f(x), xi-1 x xi,
mi * = inf h(x), xi-1 x xi,

Divide the number., 1,2....,n in two

classes A and B as follows,

i A if M i m i < and i B if M i

m i .

For i A our choice of shows that

M* i m i * < .

For i B, M i * m i * 2K, where K =

sup||(t)|, m t M.
We have (M m )
i i i i
iB iB

= M m
i i i i
iB iB

<U(P,f,) L(P,f,)

< .(by(1))
2

Hence < i ---------------------(2)


iB

U(P,f,) L(P,f,) = (M * m * ) + (M * m * )
i i i i i i
iA iB

< + 2K i i
iA iB

[
< (xn) (x0) + 2K(by(2)) ]
<[(b) (a)] + 2K

<[(b) (a) + 2K]

Since >0 is arbitary,we see that h () on [a,b]

CYP QUESTIONS:

1. Suppose increases on [a,b], a

x 0 b, is continuous at x 0 ,

f(x 0 ) = 1, and f(x) = 0 if x x 0 .

Prove that f () and that

fd = 0

Double click this page to view clearly


2. If f(x) = 0 for all irrational x, f(x)

= 1 for all rational x, prove that

f ()on [a,b] for any a < b.

SECTION-1.2 PROPERTIES OF
THE INTEGRAL

Theorem 1.2.1:
a. f1 () and f2 () on [a,b], then f1 + f2 (), cf () for
b b b b

every constant c,and (f 1 + f2) d = f 1 d + cfd = cfd


a a a a

b b

b. If f1(x) f2(x)on [a,b],then f 1 d f d. 2


a a

c. If f () on [a,b] and if a<c<b,then f () on [a,c]


c b b

and on[c,b], and fd+fd=fd


a c a

d. If f () on [a,b] and if |f(x)| M on [a,b],then


b

| fd| M[(b) (a)].


a

e. If f (1) and f (2), then f (1+2) and


b b b

fd( + )=fd +fd :


1 2 1 2
a a a

If f () and c is a positive constant,then f (c) and


b b

fd(c) = c fd
a a

Double click this page to view clearly


Proof:

If f = f 1 + f 2 and P is any partition of

[a,b], then we have

L(P,f1, ) + L(P,f2, ) L(P,f, ) U(P,f,) U(P,f1, ) + U(P,f2, ).! .................(1)

Let > 0 be given.

If f 1 (), then there exists a

partition P 1 such that

U(P1,f1,) L(P1,f1, ) < . ---------(2)

Also if f 2 (), then there exists a

partition P 2 such that

U(P2,f2,) L(P2,f2,) < . --------(3)


If P is the common refinement of P1 and P2,then (2) implies

U(P,f1,) L(P,f1,) < U(P1,f1,) L(P1,f1,) (by theorem 1.1.1) < -------------------(4)
and(3) implies

U(P,f2, ) L(P,f2,) < U(P2,f2,) L(P2,f2,)(by theorem 1.1.1) < ---------------------(5)

Adding (4) and (5), we get

U(P,f1,) + U(P,f2,) L(P,f1,) L(P,f2,) < 2 ----------(6)

Now U(P,f,) L(P,f,) U(P,f1,) + U(P,f2,) L(P,f1,) L(P,f2,)(by(1)) < 2(by(6))

(i.e)U(P,f,) L(P,f,) < 2


By theorem 1.1.3 f () on [a,b]

Double click this page to view clearly


(i.e) f 1 +f 2 () on [a,b].

Now for partition P, we have


b

U(P,f1,) < f d +
1 ----------------(7)
a
b

U(P,f2,) < f d +
2 ----------------(8)
a

Adding(7) and (8), we get


b b

U(P,f1,) + U(P,f2,) < f d + +f d +


1 2
a a
b b

= f d + f d + 2
1 2
a a
b b

By (1) U(P,f,) U(P,f1,) + U(P,f2,) < f d + f d + 2


1 2
a a
b b

(i.e)U(P,f,) < f1d + f2d + 2


a a
b

But fd U(P,f,)
a

From the above two inequalities, we

get

b b b

fd < f d + f d + 2
1 2
a a a

Double click this page to view clearly


Since >0 is arbitary,we conclude that
b b b

fd f d + f d
1 2 --------------------(10)
a a a

Replace by f1 and -f1 and f2 and -f2 in (10), we get


b b b

fd f d + f d
1 2 --------------------(11)
a a a

From (10) and(11), we get


b b b

fd f d + f d
1 2
a a a
b b b

(i.e)(f1 + f2)d = f1d + f2d


a a a

(b) Let f 1 (x) f 2 (x) on [a,b].

Then f 2 (x) f 1 (x) 0.

Since is monotonically increasing in

[a,b], (b) > (a).


b

Then we have (f (x) f (x))d 0


2 1
a


b b b

(i.e)(f2 f1)d 0 f2d f d 0


1
a a a

Double click this page to view clearly



b b b b

f2d f d
1 f1d f d
2
a a a a

Similarly we can prove (c), (d) and

(e).

Theorem 1.2.2:

If f () and g () on [a,b]

then

i. fg () ;
b b

ii. |f| () and |fd| |f| d.


a a

Proof:

2
If we take (t) = t , then by theorem

1.1.8,

f ( ) f2 () -------------------(1)

f () and g () f+g (), f-g () (by theorem 1.2.1)


(f+g) 2 2
(), (f-g) () (by(1))

Double click this page to view clearly


(f+g) 2 2
+ (f-g) () (by theorem 1.2.1)

4fg ()
1
4 (4fg) () on [a,b] (by therom 1.2.)

fg () on [a,b]

If we take (t) = | t |, then by

theorem 1.1.8,

f () | f | ().
b

Choose c=1,so that c fd 0.


a

b b b b

Then | fd| = cfd = cfd |f|d, since cf |f|.


a a a a

Definition:

The unit step function I is defined by

{
0 ( x0 )
I(x) =
1 (x>0)

Double click this page to view clearly


Theorem 1.2.3:

If a < s < b, f is bounded on [a,b],

f is continuous at s,
b

and (x) = I(x-s), then fd = f(s).


a

Proof:

Consider partitions P = {x 0 ,x 1 ,x 2 ,x 3

}, where x 0 = a and x 1 = s < x 2 < x 3

=b.
Then U(P,g,)=M11+M22+M33
[ ] [ ] [
= M1 (x1) (x0) + M2 (x2) (x1) + M3 (x3) (x2) ]
[ ] [ ] [
= M1 I(x1 s) I(x0 s) + M2 I(x2 s) I(x1 s) + M3 I(x3 s) I(x2 s) ]
= M1[0 0] + M2[1 0] + M3[1 1]

(by the definition of unit step function)


=M2
ly
III L(P,f,) = m2.

Since f is continuous at s, we see that

M 2 and m 2 converges to f(s) as x 2 s.

(i.e.) U(P,f,) and L(P,f,) converges

to f(s) as x 2 s.

Double click this page to view clearly


b

Therefore fd = f(s)
a

Theorem 1.2.4:

Suppose c n 0 for n = l,2,3,...,

c n converges, {s n } is a sequence of

distinct points in (a,b), and


(x) = c I(x-s )
n n be continuous on
n=1
b

[a,b]. Then fd = c f(s ) n n


a n=1

Proof:

Since c n converges, the series


c I(x-s )
n n is also converges for
n=1

every x. (by Comparison test).

Clearly (x) is monotonic. Also (a)

= 0 and (b) = c n (by the definition

of unit step function).


Let >0 be given.Choose N so that c n < . ---------(1)


n=N+1

Put 1(x) = c I(x-s ) and (x) = c I(x-s ).


n n 2 n n
n=1 n=N+1

By the properties of the integral and

by theorem 1.2.3,

b N

fd 1 = c f(s ) n n --------------------(2)
a n=1

Since 2(b) 2(a) = c I(b-s ) c I(a-s )


n n n n
n=N+1 n=N+1

| |
b

= cn 0 = cn < .(by (1)), we have fd2 M,-----------(3)


n=N+1 n=N+1 a

where M = sup|f(x)|. (by the

properties of integral)

Since = 1 + 2 , we have

| | | |
b N b N

fd cnf(sn) = fd(1+2) c f(s )n n


a n=1 a n=1

| | | |
b b N N b N

= fd1 + fd 2 c f(s ) n n = cnf(sn) + fd 2 c f(s ) (by (2))


n n
a a n=1 n=1 a n=1

Double click this page to view clearly


| |
b

= fd2 M(by(3))
a

| |
b N

(i.e) fd c f(s )
n n M
a n=1

If we let N ,we get fd = c f(s )


n n
a n=1

Theorem 1.2.5:

Assume increase monotonically and

' () on [a,b]. Let f be a bounded

real function on [a,b]. Then f ()

if and only if f' (). In that case


b b

fd = f(x) '(x)dx
a a

Proof:

Let > 0 be given.

Since ' (), by theorem 1.1.3,

there exist a partition

P = {a = x 0 ,x 1 ,x 2 ,...,x n =b} such that

U(P,') L(P,') < . -----------(1)

Double click this page to view clearly


By the mean value theorem,i = (xi) (xi-1) = (ti)xi, -------(2)

where ti [xi-1, xi] for i=1,2,.....n.


n

If si [xi-1, xi], then | (s ) (t )| x < .


'
i
'
i i ------------(3) (by (1) and theorem 1.1.4(b))
i=1

Put M=sup|f(x)|. -----------------------------(4)


n n

Since f(s ) = f(s ) '(t ) x (by(2)), we have


i i i i i
i=1 i=1

| | | |
n n n n

f(si) i f(s ) (s )x
i i i i = f(si) '(ti)xi f(s ) '(s )x
i i i
i=1 i=1 i=1 i=1

| |
n n

= (
f(si) '(ti) '(si) xi ) |f(s )| | '(s ) '(t )|x
i i i i
i=1 i=1

= M (by (3) and (4))

In particular
n

f(s ) U( P,f') + M,for all choices of s [x


i i i i-1, xi], so that
i=1

U(P,f,) U(P,f') + M --------------------(5)


Similarly we can show that U( P,f') U(P,f,) + M ------------(6)

Therefore |U(P,f,) U( P,f')| M ------------(7)

If P is replaced by any refinement,

then (1) is true and hence (7) is

also true. Therefore

| |
b
b _

fd-f(x)'(x)dx M.
a
a

Double click this page to view clearly


b b
_ _

Since >0 is arbitary, fd = f(x)'(x)dx


a a

for any bounded f.

b b

Similarly fd = f(x)'(x)dx for any bounded f.



a a


b b
_ b _ b

f () fd = fd f(x)'(x)dx = f(x)'(x)dx
a a
a a


b b

f ' () and fd = f(x) '(x)dx


a a

Theorem 1.2.6:

(Change of Variables) Suppose

is a Strictly increasing continuous

function that maps an interval [A,B]

onto [a,b]. Suppose a is

monotonically increasing on [a,b]

and f () on [a,b]. Define and

g on [A,B] by (y) = ((y)), g(y) =


f((y)) . Then

Double click this page to view clearly


b b

g () and gd = fd
a a

Proof:

To each partition P={x 0 , x 1 , x 2 ,...,

x n } of [a,b] corresponds a partition

Q={y 0 , y 1 , y 2 ,..., y n } of [A,B], so

that x i = (p(y i ). All partitions of

[A,B] are obtained in this way. Since

the values taken by f on [x i1 ,x i are

exactly the same as those taken by g

on [y i1 ,y i ], we see that U(Q,g, ) =

U(P,f,), L(Q,g, ) = L(P,f,).

(U(Q,g,) = Mi * i where Mi * = sup g(x), (xi-1 x xi)


i=1

= sup f((y)) = sup f(x) = Mi and i = (yi) (yi-1)

( ) ( )
= (yi) (yi-1) = (yi) (yi-1) = i

Therefore U(Q,g,) = U(P,f,))

Double click this page to view clearly


Since f (), P can be chosen so

that both U(P,f,) and L(P,f,) are


b

close to fd Hence U(Q,g, ) and


a

L(Q,g, ) are also close and by

theorem 1.1.3, g ( ) and


b b

g () and gd = fd
a a

CYP QUESTIONS:

1. Prove theorem 1.2.1 (c) , (d) and

(e).

SECTION-1.3 INTEGRATION
AND DIFFERENTIATION

Theorem 1.3.1:

Let f () on [a,b]. For a x b,


b

put F(x) = f(t)dt


a
Then F is continuous on [a,b]. Also

if f is continuous at x 0 of [a,b], then

F is differentiable at x 0 and F(x 0 ) =

f(x 0 ).

Proof:

Since f () , f is bounded.

Then there exist a real number M

such that | f(t) | M for a x b.

| |
y x

If a x y b, then |F(y) F(x)| = f(t)dt- f(t)dt


a a

| |
a y

= f(t)dt+ f(t)dt
x a

| |
y y

= f(t)dt |f(t)|dt
x x

M(y-x).

Given >0 we have |y-x| <



M |F(y) F(x)| <

Double click this page to view clearly


Therefore F is uniformly continuous

on [a,b].

F is continuous on [a,b].

Suppose f is continuous at a point x 0 .

Given > 0, choose > 0 such that

| f(t) F(x 0 ) | < if |t x 0 |< and

a t b.

Therefore, if x 0 < s < x 0 < t < x 0

+ and a s < t b

| |
t

| | (f(u) f(x ))du


F(t)-F(s)
f(x0) =
1
t-s t-s 0
s


1
t-s (f(u) f(x ))du
0
s
t

< t-s
1
du
s

<
Hence F'(x0) = f(x0).

Double click this page to view clearly


Theorem 1.3.2:

(The Fundamental theorem of

Calculus)

If f () on [a,b] and if there is a

differentiable function F on [a,b]

such that F= f then


b

f(x) dx=F(b) F(a)


a

Proof:

Let > 0 be given.

Choose a partition P = {x 0 x 1

x 2 ....x n } of [a,b] so that

U(P,f) L(P,f) < .

By the Mean Value theorem, there

are points t i [x i1 ,x i such that


F(x i ) F(x i1 ) = f(t i )x i for i =

1,2,...., n.
n

Thus f(t ) x = F(b) F(a).


i i
i=1

It follows from theorem 1.1.4 (c) that

| |
n b

f(t )x fd
i i <
i=1 a

| |
b

(i.e) (F(b) F(a)) fd <


a

Since this is hold for every > 0, we

have

f(x)dx = F(b) F(a)


a

Theorem 1.3.3:

(Integration by Parts)
Suppose F and G are differentiable

function on [a,b], F= f and


b b

G'=g then F(x)g(x)dx=F(b)G(b) F(a)G(a) f(x)G(x)dx


a a

Proof:

Put H(x) = F(x)G(x), x [a,b].

Then H'(x) = F'(x)G(x) + F(x)G'(x)

= f(x)G(x) + F(x)g(x)

F= f and G= g H on

[a,b].

Let us apply the Fundamental

theorem of Calculus to the function

H and its derivative H, we get


b

H'(x)dx=H(b) H(a)
a

Double click this page to view clearly


b

(f(x)G(x) + F(x)g(x))dx=F(b)G(b) F(a)G(a)


a
b b

f(x)G(x)dx+F(x)g(x)dx = F(b)G(b) F(a)G(a).


a a
b b

(i.e)F(x)g(x)dx=F(b)G(b) F(a)G(a) f(x)G(x) dx.


a a

Hence the theorem.

CYP QUESTIONS:

1. Suppose f 0, f is continuous on
b

[a,b], and f(x) dx=0 Prove that


a

f(x) =0 for all x [a,b].

Section 1.4 Integration of


vector valued functions

Definition:

Let f 1 ,f 2 ,...,f k be a real functions on

[a,b], and let

Double click this page to view clearly


f = (f 1 ,f 2 ,...,f k ) be the corresponding
k
mapping of [a,b] into R . If

increases monotonically on [a,b], to

say that f () means that f j

() for j = 1,2,....,k. If this is the

case we define

( )
b b b

fd = f d, ...., f d
1 k
a a a

Theorem 1.4.1:

(Analogue of the Fundamental

theorem of Calculus for vector


valued functions)

k
If f and F map [a,b] into R , if f

on [a,b] and if F' = f then

f(t)dt =F(b) F(a)


a
Proof:

Let f = (f 1 ,f 2 ,... ,f k ) and F = (F 1

,F 2 ,... ,F k ).

F = f (F1, F2, ..., Fk) = (f1, f2, ..., fk).


Fj ' = fj, j = 1, 2, ...., k.

By the Fundamental theorem of

Calculus, for real valued function f j ,

we have f (t)dt = F (b) F (a)


j j j
a

( )
b b b

(i.e) f1(t)dt, f (t)dt, ....., f (t)dt


2 k
a a a

=(F1(b) F1(a), F2(b) F2(a), ...., Fk(b) Fk(a))


b

(i.e.)f(t)dt=F(b) F(a)
a
Theorem 1.4.2:

k
If f maps [a,b] into R and if f ()

for some monotonically increasing

function on [a,b] then | f | ()

and

| |
b b

fd |f|d
a a

Proof:

Let f = (f 1 ,f 2 ,...,f k ). Then | f | =


2 2 2
(f 1 +f 2 +.. .+f k )

f ( ) f1, f2, ...fk () on [a,b]


f12, f22, ...., fk2 () on [a,b]
f12+f22 + ... + fk2 () on [a,b]

(f12+f22 + .... + fk2)


1/2
() on [a,b]

|f| () on [a,b]

Double click this page to view clearly


b

Put y=(y1, y2, ...yk), where yj = f d for j=1,2,....k.


j
a
b k k b

fd and |y| y .y = y f d
2
Then y= = j j j j
a j=1 j=1 a
b b

= y f d |y||f| d
j j
a a
b

=|y| |f| d.
a

If y = 0, the inequality is trivial.

If y 0, then divide by | y |, we get

| |
b b b

|y| |f|d. That is fd |f| d


a a a

CYP QUESTIONS:
b

1. Define fd if f:[a,b] Rk
a

Double click this page to view clearly


Section 1.5 Rectifiable curves

Definition:

A continuous mapping of an interval


k k
[a,b] into R is called a curve in R .

If is 1 1 , then is called an arc.If

(a) = (b), is said to be a closed

curve.

We associate to each partition P =

{x 0 , x 1 , X 2 ,..x n }of [a,b] and to each

curve on [a,b], the number


n

(P,) = |(x ) (x )|
i i-1
i=1

Here (P, ) is the length of the

polygonal path with vertices at (x 0 ),


(x 1 ),...,(x n ). The length of is

defined as () = sup (P, ).

If () < , we say that is

rectifiable.

Theorem 1.5.1:

If is continuous on [a,b] , then is


b

rectifiable and () = |'(t)dt


a

Proof:

If a x i1 x i b, then |(x i ) (

x i1 )|

| |
xi xi

= '(t)dt |'(t)| dt
xi-1 xi-1

Hence (P,) |'(t)| dt for every


a

partition P of [a,b].
b

We have sup (P,) |'(t)| dt


a

(i.e.) () |'(t)| dt -----(1)


a

Now to prove the reverse inequality.

Let > 0 be given.

Since is continuous on [a,b], it is

uniformly continuous on [a,b].

> 0 such that | s t | < |

(s) (t)| < .

Let P = P = {x 0 , x 1 , x 2 ,...,x n } be a

partition of [a,b] with x i < , i

If x i1 t x i , it follows that | (t)

(x i )| < .

(i.e.) | (t)| | (x i )| < .

(i.e.) | (t)| | (x i )| +.
xi xi xi

Hence |'(t)|dt |'(x ) + | dt (|'(x )| + )dt


i i
xi-1 xi-1 xi-1

(|'(x )| + )(x x
i i i-1 )

(|'(x )| + )x
i i

|
'(xi) xi + xi |

|( |
xi

)
'(t) + '(xi) '(t) dt + xi
xi-1

| | |( ) |
xi xi

'(t)dt + '(xi) '(t) dt


xi-1 xi-1

| |
xi

| | (
(xi) (xi-1) +) '(xi) '(t) dt
xi-1

xi

|
(xi) (xi-1) + | dt + xi
xi-1

| |
(xi) (xi-1) + (xi + xi-1)xi

|
(xi) (xi-1) + 2xi |
If we add this inequality for i = 1,2,..

.,n,

n xi n n

we get |'(t)| dt |(x ) (x )|+2 x i i-1 i


i=1 xi-1 i=1 i=1

xn

(i.e.)|'(t)| dt (P,) + 2(xn x0)


x0

Double click this page to view clearly


b

|'(t)| dt (P,) + 2(b-a)


a

Since > 0 is arbitrary,we have


b

|'(t)| dt (P,).
a

Hence |'(t)| dt (). --------------------(2)


a
b

From (1) and (2), we get () = |'(t)| dt


a

CYP QUESTIONS:

1. Suppose f is a bounded real


2
function on [a,b] , and f on

[a,b] , Does it follow that f

? Does the answer change if we


3
assume that f ?

2. Let P be the Cantor set. Let f be


a bounded real function on [0,1]

which is continuous at every


2
point outside P. Prove that f f

on [0,1].

Double click this page to view clearly


UNIT-2

Unit Structure

Section 2.1: Discussion of the

main problem

Section 2.2: Uniform

convergence

Section 2.3: Uniform

convergence and continuity

Section 2.4: uniform

convergence and integration

Section 2.5: uniform

convergence and Differentiation

Section 2.6: Equi continuous

families of functions

Section 2.7: The Stone

Weierstrass Theorem
Introduction

In this unit we confine our attention

to complex valued functions,

although many of the theorems and

proofs which follow extend without

difficulty to vector-valued functions,

and to mappings into general metric

spaces. We discuss the concept of

uniform convergence and continuity,

differentiation and integration.

SECTION-2.1 DISCUSSION OF
THE MAIN PROBLEM

Definition:

Suppose {f n }, n = 1,2,..,, is a

sequence of functions defined on a


set E, and suppose that the sequence

of numbers {f n (x)} converges for

every x E. We can then define a

function f by

f(x) = lim fn(x)( xE ). -------------( 1 )


n

{f n } converges to f point wise on E if

(1) holds.

Similarly, if f n (x) converges for

every x E, and if we define


f(x) = f (x)( xE ) the function f is


n
n=1

called the sum of the series fn

To say that f is continuous at x

means lim f(t) = f(x) Hence to ask


t x

whether the limit of a sequence of


continuous functions is continuous is

the same as to ask whether

lim lim f(t) = lim lim f(t) (i.e.)


t x n n t x

whether the order in which limit

processes are carried out is

immaterial. On the LHS , we first let

n then tx; on the RHS tx first,

then n.

We shall now show by means of

several examples that limit processes

cannot in general be interchanged

without affecting the result.

Example 1:

For m = 1,2,...,n = 1,2,..., let


m
sm,n = m+n
Then , for every fixed n,
1
lim sm,n = lim 1 + n/m =1 so that
m m

lim lim sm,n = 1


n m

On the other hand, for every fixed


m
m, lim sm,n = n(m+1) = 0. so that
n

lim lim sm,n = 0.


n n

Example 2:

2
x
Let fn(x) = 2 n (x real; n=0,1,2,...),
( 1+x )

2
x
and consider fn(x) = fn(x) = 2 n
. -------------(1)
n=0 n=0 ( 1+x )

Since f n (0) = 0, we have f(0) = 0.

For x 0, the last series in (1) is

convergent with sum (?).

Double click this page to view clearly


{
0 (x=0)
Hence f(x) = , so that
2
1+x ( x0 )

a convergent series of continuous

functions may have a discontinuous

sum.

Example 3:
2n
For m=1,2,...,put fm(x) = lim (cosm ! x)
m

When m!x is an integer , f m (x) = 1.

For all other values of x, f m (x) = 0.

Now let f(x) = lim fm(x)


m

For irrational x, f m (x) = 0 for every

m; hence f(x) = 0.

For rational x, say x = p/q, where p

and q are integers, we see that m!x


is an integer if m q, so that f(x) =

1.

{
2n
0 (x irrational)
Hence lim lim (cosm! x) =
m n 1 (x rational)

(i.e.) an everywhere discontinuous

limit function, which is not Riemann-

integrable.

Example 4:

sin nx
Let fn(x) = n (x real;n=1,2,3,...), and

f(x) = lim fn(x) = 0


n

Then f (x) = 0, and fn '(x) = n cos nx

, so that {f n } does not converge

to f . For instance
fn '(0) = n + as n , whereas

f(0) = 0.

Example 5:

2 n
2
(
Let fn(x) = n x 1 x ) (0 x 1,n=1,2,3....) --------(2)
For 0 x 1,we have lim fn(x) = 0
n

( )

n
Since,if p>0 and is real,then lim n =0
(1+p)
n

Since fn(0) = 0, we see that lim fn(x) = 0 (0 x 1). ------------(3)


n

Also ( x 1x
2 n
) dx= 2n+2
1
.
0


2
n
fn(x)dx= 2n+2 + as n
0

2
If we replace n by n in (2), (3) still

holds, but we have

[ ]
1 1

lim
n 0
f (x)dx= lim
n
n
n
2n+2
1
= 2 , whereas lim fn(x) dx=0.
n
0
The limit of the integral need not

be equal to the integral of the limit,

even if both are finite.

CYP QUESTIONS:

1. Give more examples for the limit

of the integral need not be equal

to the integral of the limit, even

if both are finite.

SECTION-2.2 UNIFORM
CONVERGENCE

Definition:

A sequence of functions {f n }, n =

1,2,3,...., is said to converge

uniformly on E to a function f if for

every > 0 there is an integer N such

that n N implies |f n (x) f(x)|

for all xE.---(1)


Note 1: Every uniformly convergent

sequence is point wise convergent.

Note 2: The difference between

uniform convergent and point wise

convergent is : If {f n }converges

point wise on E, then there exists

a function f such that, for every

> 0 and for every xE, there is an

integer N , depending on and on

x, such that (1) holds if n N. If

{f n } converges uniformly on E, it is

possible , for each > 0, to find one

integer N which will do for all xE.

Note 3: We say that the series f n

(x) converges uniformly on E if the

sequence {s n }of partial sums defined


n

by f (x) = s (x)
i n converges
i=1

uniformly on E.
Cauchy criterion for uniform

convergence:

Theorem2.2.1 :

The sequence of functions {f n },

defined on E, converges uniformly on

E if and only if for every > 0 there

is an integer N such that n N, m

N, xE implies |f n (x) f m (x)| .

Proof:

Suppose {f n } converges uniformly on

E.

Let f be the limit function and let >

0.

Then there is an integer N such that

n N, xE implies
|fn(x) fm(x)| 2

Therefore |fn(x) fm(x)| |fn(x) f(x)| + |f(x) fm(x)|



<2 + 2 = if n N,m N, x E

Conversely, suppose the Cauchy

condition holds.

(i.e.) for every > 0 there is an

integer N such that n N, m N,

xE implies

|fn(x) fm(x)| -----(1)

By a theorem, (f n (x)} converges, say

to f(x) for every x ( since R is

complete).

Thus the sequence {f n } converges on

E, to f.
We have to prove that the

convergence is uniformly.

Fix n, and let m in(1).

Since f m (x) f(x) as m , this

gives |f n (x) f(x)| for every n

N and every xE.

{f n } converges uniformly to f on E.

Theorem 2.2.2:

Suppose lim fn(x) = f(x) ( x E )


n
.
Put Mn = sup |fn(x) f(x)|
xE

Then f n f uniformly on E if and only

if

M n 0 as n .
Proof:

Suppose f n f uniformly on E.

By definition, for every > 0 there is


an integer N such that n N implies

|f n (x) f(x)| for all xE.

Therefore sup |fn(x) f(x)|


xE

(i.e.) M n if n N.

(i.e) M n 0 as n.

Conversely, suppose M n 0 as n

Then, given > 0 there is an integer

N such that n N M n .

(i.e.) n N
sup |fn(x) f(x)|
xE

|f (x) f(x)| for every x E.


n
Therefore fn f uniformly on E.

Weierstrass theorem on

uniform convergence.

Theorem 2.2.3:

Suppose {f n }is a sequence of

functions defined on E, and suppose

|f n (x)| M n (xE, n = 1,2,3,....).

Then f n convergence uniformly on E

if M n converges.

Proof:

Suppose M n converges.

Then, for arbitrary > 0,

| |
m m m

f (x)
i |f (x)| M
i i ( xE )
i=n i=n i=n

(xE) provided m and n are large

enough.
(i.e.) there is an integer N such that

n N, m N, xE implies

|f n (x) f m (x)| .

By theorem 2.2.1, f n convergence

uniformly on E.

CYP QUESTIONS:

1. Prove that every uniformly

convergent sequence of bounded

functions is uniformly bounded.

SECTION-2.3 UNIFORM
CONVERGENCE AND CONTINUITY

Theorem 2.3.1:

Suppose f n f uniformly on a set E in

a metric space.Let x be a limit point

of E, and suppose that lim fn(t) = An


t x
(n = 1,2,3,....) . Then

{A n }converges, and

lim fn(t) = lim An


t x n

In other words, the conclusion is that

lim lim fn(t) = lim lim fn(t)


t x n n t x

Proof:

Let > 0 be given.

Since {f n }converges uniformly on E,

there exists N such that n N,

m N,t E|fn(t) fm(t)| ---------------------(1)

By hypothesis,for n=1,2,3,......, lim fn(t) = An --------(2)


tx

Letting t x in (1) and using (2), we

get n, m N implies |A n A m |

Therefore {A n }is a Cauchy sequence

in the set of real numbers R.


Since R is complete, {A n } converges

to some point, say, A.

|f(t) A| |f(t) fn(t) + fn(t) An + An A|


|f(t) fn(t)| + |fn(t) An| + |An A| ----------------(3)

Since f n f uniformly on E, choose a

positive integer n such that

|f(t) fn(t)| 3 , for all t E



--------------------(4)

Since An A,we have m N implies |An A| 3 ---------(5)

Then, for this n, we choose a

neighborhood V of x such that

|f(t) An| 3

(
if t V E,t x. since lim
tx
fn(t) = An
) -----(6)

Using (4),(5) and (6) in (3),we get |f(t) A|



3 + 3 + 3 =

if t V E,t x

(i.e.) lim f(t) = A= lim An


t x n

(i.e) lim lim fn(t) = lim lim fn(t)


t x n n tx

Double click this page to view clearly


Theorem 2.3.2:

(Corollary to theorem 2.3.1) If {f n }

is a sequence of continuous functions

on E, and if f n f uniformly on E,

then f is continuous on E.

Proof:

Since {f n } is a sequence of

continuous functions on E, for every

n, we have lim fn(t) = fn(x)


t x

By theorem 2.3.1, we have

lim lim fn(t) = lim lim fn(t)


t x n n t x

t x n
( )
(i.e.) lim fn(t) = lim fn(x) = f(x) since fnf uniformly on E .

(i.e) lim f(t) = f(x)


t x

By definition of continuous function, f

is continuous on E.

Double click this page to view clearly


Note:The converse of the above

theorem is need not be true.

Example:

2 2 n
f n (x) = n x(1 x ) (0 x 1, n =

1,2,3,....)

Theorem 2.3.3:

Suppose K is compact, and

a. {f n } is a sequence of continuous

functions on K,

b. {f n }converges point wise to a

continuous function f on K,

c. f n (x) f n+1 (x) for all x K, n=

1,2,3,.....

Then f n f uniformly on K.

Proof:

Put g n = f n f.
Since f n and f are continuous, g n is

also continuous.

Since f n f point wise , g n 0 point

wise.

Also , since f n (x) f n+1 (x) for all

xK, f n (x) f(x) f n+1 (x) f(x), for

all xK g n (x) g n+1 (x) for all xK.


We have to prove that fn f

uniformly on K.
(i.e.) to prove that g n 0 uniformly

on K.

Let > 0 be given.

Let K n = {xK \ g n (x) .}

(i.e.) K n = {xK \ g n (x) [,)}.

1
(i.e.) K n = {xK \ xg n ([,))}.

1
(i.e.) K n = g n ([,)).
Since g n is continuous and [,) is

closed, K n is closed ,and hence K n

is compact (since closed subsets are

compact).

Let xK n+1 . Then g n+1 (x) .

Since g n (x) g n+1 (x) , x K n .

Then K n K n+1 n.

Fix xK. Since g n (x) 0 point wise,

we see that x K n if n is sufficiently

large. Thus x K n .

In other words, K n is empty.

K n is empty for some N. It follows

that 0g n (x)<,x K&n N.

Therefore | g n (x) 0| < for all x K

and for all n N.


(i.e.) g n 0 uniformly on K.

(i.e.) f n f uniformly on K.

Definition:

If X is a metric space, C(X) will

denote the set of all complex valued,

continuous, bounded functions with

domain X.

C(X) consists of all complex

continuous functions on X if X is

compact.

We associate with each f C(X) its

supremum norm ||f|| = sup |f(x)|.

xE

Since f is bounded, ||f|| < . Also

||f|| = 0 f(x) = 0 for every xX


(i.e.) ||f|| = 0 f=0.

If h = f + g, then |h(x)| =

|f(x)+g(x)| |f(x)|+|g(x)|

||f||+||g|| for all xX Hence ||f+g||

= ||h|| ||f||+||g||.

Also C(X) is a metric space with the

metric d(f,g) = ||f g||.

Theorem 2.3.4:

C(X) is a complete metric space.

Proof:

Let {f n }be a Cauchy sequence in C

(X).

to. each > 0, there exists a

positive integer N such that n, m N

implies ||f n f m || < .


It follows that there is a function f

with domain X to which {f n }

converges uniformly, (by Cauchy

criterion for uniform convergence).

By theorem 2.3.2, f is continuous.

Since f n is bounded and there is an

n such that |f(x) f n (x)| <1 for all

xX, f is bounded.

Thus f C(X).

Since f n f uniformly on X, we have

||f f n || 0 as n .

CYP QUESTIONS

1. If {f n } and {g n } converge

uniformly on a set E, prove that

{f n + g n } converges uniformly on

E.
2. If {f n } and {g n } converge

uniformly on a set E and if {f n }

and {g n }are sequences of

bounded functions, prove that

{f n g n } converges uniformly on E.

SECTION-2.4 UNIFORM
CONVERGENCE AND INTEGRATION

Theorem 2.4.1:

Let be monotonically increasing on

[a,b]. Suppose f n ().on [a,b], for

n = 1,2,3,...., and suppose f n f

uniformly on [a,b].Then f,and


b b

fd = n
lim f d
a
n
a

Proof:

Put n = sup |f n (x) f(x)|, the

supremum being taken over a x

b.
(i.e) n f-fn n. -----------------(1)

(i.e.)fn n f fn + n
b b
b b _ _

(f n n) d fd fd (f n + n)d
a a
a a
b
b b _ b

(f n n) d fd fd (f n + n)d
a a a
a
b
_ b b b

0 fd fd (f n + n)d- (f - )d n n
a a a
a
b b

(f n + n fn + n)d = 2 d=2 [(b) (a)] -----(2)


n n
a a

By theorem 2.2.2,n
0 as n
( . fn f uniformly on [a,b])
b
_ b

From(1), 0 fd fd 0 as n
a
a
b
_ b

Therefore fd = fd -----------(3)
a
a

(i.e.) f().[a,b].

b b b b

Using(3) in (1), we get- d fd f d d


n n n
a a a a
b b

n[(b) (a)] fd f d [(b) (a)] n n


a a

| |
b b

(i.e) fd f d n n[(b) (a)] 0 as n


a a

b b

Therefore fd = n
lim
a
f d. n
a

Double click this page to view clearly


CYP QUESTIONS

1. If fn ().on [a,b] and if


f(x) = f (x)(a x b),


n the
n=1

series converging uniformly on


b b

[a,b], then fd = f d n
a n=1 a

SECTION-2.5 UNIFORM
CONVERGENCE AND
DIFFERENTIATION

Theorem 2.5.1:

Suppose {f n } is a sequence of

functions, differentiable on [a,b] and

such that {f n (x 0 )} converges for

some point x 0 on [a,b]. If {f n '}

converges uniformly on [a,b], then

{f n } converges uniformly on [a,b],


to a function f, and

f '(x) = lim fn '(x)(a x b)


n

Proof:

Let > 0 be given.

Since {f n (x 0 )}converges for some

point x0 on [a,b], and every

convergent sequence is Cauchy ,

choose N such that n N,

|
m N,t E implies fn(x0) fm(x0) |
2 ------------------(1)

Also, since {f n '} converges uniformly

on [a,b], say to f, we have

|fn '(t) fm '(t)| < 2(b-a) (a t b)



------------------(2)

Apply Mean Value theorem to the

function f n f m , we get

Double click this page to view clearly


|(fn fm)(x) (fn fm)(t)| = |fn(x) fn(t) + fm(t)|
|
= (fn(x) fn(t)) + (fm(x)-fm(t)) |
|x-t||fn '(t) fm '(t)| (by MVT)

<|x-t| 2(b-a) (by(2))



------------(3)

2 .for any x and t on[a,b], if n,m N -------------(4)

|
Also |fn(x) fm(x)| = fn(x) fm(x) fn(x0) + fm(x0) + fn(x0) fm(x0) |
| ||
fn(x) fm(x) fn(x0) + fm(x0) fn(x0) fm(x0) |
(by(1) and (4))

<2 + 2

=,for any x on [a,b],if n,m N

Therefore {fn} converges uniformly on [a,b]

Let f(x) = lim fn(x)(a x b)


n

Let us now fix a point x on [a,b] and


fn(t) fn(x)
define n(t) = t-x
f(t) f(x)
( t) = t-x for a t b,t x.

Allowing n
in n(t), we get lim n(t) = lim
n n
fn(t) fn(x)
t-x ,

lim fn(t) lim fn(x)


n n f(t) f(x)
t-x = t-x = (t). -----------(5)
fn(t) fn(x)
Also lim n(t) = lim t-x = fn '(x) -----------(6)
tx t-x

f(t) f(x)
and lim (t) = lim t-x = f '(x), for a t b,t x --------------(7)
tx t-x

The inequality (3)can be rewritten as

| |<
fn(x) fn(t) fm(x) + fm(t)
t-x 2(b-a)

| |<
fn(x) fn(t) fm(x) fm(t)
(i.e) t-x t-x 2(b-a)

(i.e.)|n(t) m(t)| <



2(b-a)
.

Double click this page to view clearly


The above equation shows that { n }

converges uniformly to for t x.

Apply theorem 2.3.1 to { n }, we get

lim lim n(t) = lim lim n(t).


t-x n n t-x

(i.e) lim n(t) = lim fn '(x) (by(5) and (6)).


t-x n

f '(x) = lim fn '(x) (by(7)).


n

Theorem 2.5.2:

There exists a real continuous

function on the real line which is

nowhere differentiable.

Proof:

Define (x) = |x| ( 1 x 1) -------------(1)

Extend the definition of (x) to all

real x by requiring that

(x+2) = (x) -----------------------------(2)

Double click this page to view clearly


| |
Then,for all s and t,we have (s) (t) = |s| |t| |s-t|. ------(3)

1
In particular, is continuous on R .

(?).

( ) (4 x)
n
3 n
Define f(x) =
4
n=0

| ( ) ( ) | | ( ) |

n n
3 3
Since 0 1,we have |f(x)| =
n
4 x
4 4
n=0 n=0

( )
n
3

4
n=0

( )
n
3
Since is a geometric series
4
n=0
3
with the common ratio 4 < 1 and

( )
n
3 1
hence converges in R .
4
n=0

By theorem 2.2.3, the series


( )
n
3
4
4 x ( ) n
converges uniformly
n=0
1
on R . By theorem 2.3.2, f is
1
continuous on R .

Double click this page to view clearly


Fix a real number x and a positive
1 -m
integer/m.m = 2 .4 where the

sign is so chosen that no integer lies


m m
between 4 x and 4 (x + m ). This
m
can be done , since | 4 (x + m )
m
4 x |

| m
= 4 m = 4 | m
|m| = 4
m
| 12 4-m| = 4m4-m 12 = 12
(
4 (x+m) 4 x
n
) (n)
Define m = m

n
When n > m, 4 m = 4 m = 4 2 .4
n n
( 1 -m
) = 12 4n-m = 12 22(n-m) = 22(n-m) 1
= even integer.

n n
Therefore (4 (x + 5 m )) (4 x)

= 0. (i.e.) n = 0 when n > m.

| |
f(x+m) f(x)
We conclude that m

| |

| ( ) |
( ) ) ( ) (4 x)
n n
3 3
4 (
4 x+m
n
4
n

n
3
= = n
n=0 n=0
m 4
n=0

| ) |(since, = 0 when n > m)


m

(
n
3
= n n
4
n=0

m-1 m-1

( ) ( ) (by (5))
n n
3 m 3 3 m m 3
() 4 m
4
n () 4 4
4 n
n=0 n=0

Double click this page to view clearly


m-1

3
m
3 n m
( 2
3 1 + 3 + 3 + .... + 3
m-1
)
n=0
m
( m
)
( ) ( )
m 3 1
m
m
m
3 1 2.3 3 1
3 3 12 3 2 2

( ).
m
3 +1
2

As m ,m 0

It follows that f is not differentiable

at x.

SECTION-2.6
EQUICONTINUOUS FAMILIES OF
FUNCTIONS

Definition:

Let {f n } be a sequence of functions

defined on a set E. We say that {f n }

is point wise bounded on E if the

sequence {f n (x)}is bounded for every

xE, that is, if there exists a finite-

valued function defined on E such

Double click this page to view clearly


that |f n (x)| < (x) (xE, n =

1,2,3,....)

We say that {f n }is uniformly bounded

on E if there exists a number M such

that |f n (x)| < M (xE, n = 1,2,3,....)

Note: If {f n }is uniformly bounded

sequence of continuous functions on

a compact set E, there need not exist

a subsequence which converges point

wise on E.

Example:

Let f n (x) = sin nx (0 x 2, n =

1,2,3,....)

Suppose there exists a sequence

{n k }such that {sin n k x} converges,


for every x [0,27]. In that case,

we have

lim (sin nkx-sin nk+1x) = 0, (0 x 2) and hence


k
2
lim (sin nkx-sin nk+1x) = 0, (0 x 2)
k

By Lebesgue's theorem,
2

(sin n x-sin n
2
lim k k+1x ) dx=0 --------(1)
k 0

But
2

(sin n x-sin n
2
k k+1x ) dx = 2, which contradicts (1)
0

Note: Every convergent sequence

need not contains a uniformly

convergent subsequence.

For example
2
x
fn(x) = 2 2 (0 x 1,n=1,2,3,.....)
x + (1 nx)

Then | f n (x) | 1 so that {f n }is

uniformly bounded on [0,1].


Also lim fn(x) = 0, (0 x 1)
n

But f n (l/n) =1, (n = 1,2,3,....), so

that no subsequence can converge

uniformly on [0,1].

Definition:

A family of complex functions f

defined on a set E in a metric space

X is said to be equicontinuous on E if

for every > 0 there exists a > 0

such that |f(x) f(y)| < whenever

d(x,y) < , xE, yE, and f .Here

d denotes the metric of X.

Note: Every member of an

equicontinuous family is uniformly

continuous.
Theorem 2.6.1:

If {f n } is a point wise bounded

sequence of complex functions on a

countable set E, then {f n } has a

subsequence {f nk } such that {f nk

(x)} converges for every xE .

Proof:

Let {x i }, i = 1,2,3,..., be the points

of E, arranged in a sequence. Since

{f n (x 1 )}is bounded, there exists a

subsequence , which we shall denote

{f 1,k }, such that {f 1,k (x 1 )}

converges as k .

Let us now consider sequences S 1 ,

S 2 , S 3 ,.....,which we represent by the

array
S1: f1,1f1,2f1,3f1,4.... ....
S2: f2,1f2,2f2,3f2,4.... ....
S3:f3,1f3,2f3,3f3,4.... ....
.... ........ ........ ............
and which have the following

properties:
a.
S n is a subsequence of S n1 , for
n = 2,3,4,......

b. {f n,k (x n )} converges as k .

c. The order in which the functions


appear is the same in each
sequence; i.e., if one function

precedes another in S 1 , they are

in the same relation in every S n ,

until one or the other is deleted.

Hence, when going from one row

in the above array to the next

below, functions may move to

the left but never to the right.


We now go down the diagonal of

the arrays; i.e., we consider the

sequence S: f 1,1 f 2,2 f 3,3 f 4,4 .... ....

By (c), the sequence S (except

possibly its first n 1 terms) is a

subsequence of Sn, for n =

1,2,3,......Hence (b) implies that

{f n,n (x i )} converges as n , for

every x i E.

Theorem 2.6.2:

If K is a compact metric space, if

f n c(K) for n = 1,2,3,....,and if


{f n }converges uniformly on K, then

{f n } is , equicontinuous on K.

Proof:

Let > 0 be given.


Since {f n }converges uniformly, there

is an integer N such that n > N

implies ||f n f N || < . (refer

definition of ?(X) in sec 2.3).

Since continuous functions are

uniformly continuous on compact

sets, there is a > 0 such that |f i (x)

f i (y)| < if 1 i N and d(x,y) <

If n > N and d(x,y) < , it follows

that

|fn(x) fn(y)| |fn(x) fN(x) + fN(x) fN(y) + fN(y) fn(y)|


|fn(x) fN(x)| + |fN(x) fN(y)| + |fN(y) fn(y)|

fn fN + |fN(x) fN(y)| + fN fn

<++ = 3.

Therefore {f n } is equicontinuous on

K.
Theorem 2.6.3:

If K is a compact, if f n ?(K) for n

= 1,2,3,....,and if {f n } is point wise

bounded and equicontinuous on K,

then

a. {f n } is uniformly bounded on K,

b. {f n } contains a uniformly

convergent subsequence.

Proof:

a. Let > 0 be given.

Since {f n } is equicontinuous on

K, there exists a > 0 such that

x,yK
d(x,y) < |f (x) f (y)| < ,for all n.
n n -------------(1)

Since K is compact, there are

finitely many points p 1 , p 2 ,.....,


p r in K such that to every xK

corresponds at least one p i with

d(x, p i ) < .

Since {f n } is point wise bounded,

there exist M i < such that


(6) and (7) implies that h(t) < f(t) + . ( tk )
(i.e.)f(t) < h(t) < f(t) + ( tK ).
(i.e.) <h(t) f(t) < ( tK )
(i.e)|h(t) f(t)| < ( tK )

|fn(pi)| < Mi for all n. -------------------(2)

|
If M=max(M1, M2, .....Mr), then|fn(x)| = fn(x) fn(pi) + fn(pi) |
| | |
fn(x) fn(pi) + fn(pi) |
<+Mi.(by(2) & (3))

+M

Therefore, {f n } is uniformly

bounded on K.

b. Let E be a countable dense

subset of K.

By theorem 2.6.1, {f n } has a

subsequence { f ni } such that {f ni

(x)} converges for every xE .

Double click this page to view clearly


Put f ni = g i .

We shall prove that {g i }

converges uniformly on K.

Let > 0 be given.

Since {f n } is equicontinuous on

K, there exists a > 0 such that

d(x,y) < |f (x) f (y)| < , for all n.


n n ----------------(4)

{
Let V(x,) = y E/d(x,y) < . }
Since E is dense in K and K is

compact, there are finitely many

points x 1 , x 2 ,....., x m in E such

that K V(x 1 , )V(x 2 ,

)....V(x m , ).

Since {g i (x)} converges for

every xE, there is an integer N

such that

Double click this page to view clearly


|gi(xs) gj(xs)| < whenever i N,j N,1 s m. ---------(5)

If xK V(x 1 , )V(x 2 ,

)....V(x m , ), then xV(x s ,

) for some s (i.e.) d(x, x s ) <

|gi(x s ) g j (x s )| < for every

i.

If i N, j N ,
|gi(x) gj(x)| = |gi(x) gj(xs) + gi(xs) gj(xs) + gj(xs) gj(x)|
| | | | |
gi(x) gi(xs) + gi(xs) gj(xs) + gj(xs) gj(x) |

<++ = 3

Therefore, {g i } converges

uniformly on K.

(i.e.) {g i } converges uniformly

on K

(i.e.) { f ni } converges uniformly

on K

Double click this page to view clearly


(i.e.) {f n } contains a uniformly

convergent subsequence.

CYP QUESTIONS:

1. Suppose {f n }, {g n } are defined

on E, and

a. f n has uniformly bounded

partial sums;

b. g n 0 uniformly on E;

c. g 1 (x) g 2 (x) g 3 (x)

.....for every xE.

Prove that f n g n converges uniformly

on E.
SECTION-2.7 THE STONE
WEIERSTRASS THEOREM

Theorem 2.7.1:

If f is a continuous complex function

on [a,b], there exists a sequence of

polynomials P n such that lim P n (x)


x

= f(x) uniformly on [a,b]. If f is real,

the P n may be taken real.

Proof:

We may assume, without loss of

generality, that [a,b] = [0,1]. We


may also assume that f(0) = f(1) =

0. For if the theorem is proved for

this case, consider g(x) = f(x) f(0)

x[f(1) f(0)] (0 x 1).


Here g(0) = g(1) = 0, and if g can be

obtained as the limit of a uniformly

convergent sequence of polynomials ,

it is clear that the same is true for f,

since f g is a polynomial.

Furthermore, we define f(x) = 0 for

x outside [0,1]. Then f is uniformly

continuous on the whole line.

2 n
(
We put Qn(x) = cn 1 x ) (n = 1,2,3,....), ------(1)
1

where cn is chosen so that Q (x) dx=1(n=1,2,3,...) --------(2)


n
1

1 1 1 / n

Now (1 x )2 n
dx=2 (1 x ) 2 n
dx 2 (1 x ) 2 n
dx
1 0 0

1 / n

2 (1 nx ) dx(by binomial theorem)


2 n

[ ]
3 1 / n
nx
2 x- 3
0

[ ]
3
n n(1)
=2
1
n
3 [1
0 = 2 n 3n
1
]

Double click this page to view clearly


=2 [ 32 n ] = [ 34 n ] 1n


1 1

Equation (2) implies that c (1 x ) dx=1


n
2 n
cn(1 x ) dx=12 n

1 1

1>cn 1n cn < n -----------(3)

2 n
For any >0(1) and (3) implies that Qn(x) < n 1 ( )
where |x| 1 ------------------(4)

Qn(x) 0 uniformly in |x| 1.


1

Now set Pn(x) = f(x+t)Q (t)dt n ----------------(5)


1
x 1x 1

Pn(x) = f(x+t)Q (t)dt+f(x+t)Q (t)dt+f(x+t)Q (t)dt


n n n
1 x 1x

Put x + t = y. Then dx = dt & t =


1 y = x 1 , t = x y = 0,

t=1xy=1,t=1 y = x + 1.

0 1 x+1

Pn(x) = f(y) Q (y-x)dy+f(y) Q (y-x)dy+f(y) Q (y-x)dy


n n n
x-1 0 1
1

=0+ f(y) Q (y-x)dy+0( ? )


n
0

The RHS integral is clearly a

polynomial in x. Thus {P n } is a

sequence of polynomials which are

real if f is real.

Double click this page to view clearly


Therefore, given > 0, we choose

> 0 such that |y x| < implies


|f(y) f(x)| < 2 .

-----------------(6)

Let M= sup |f(x)| (since f is bounded on[0, 1]) --------(7)


x [0.1]

If 0 x 1

| |
1 1

|Pn(x) f(x)| = f(x+t)Qn(t)dt-f(x) Q (t)dt (by(2) & (5))


n
1 1

| |
1

= (f(x+t) f(x))Qn(t)dt
1

|(f(x+t) f(x))Q (t)dt| n


1

|(f(x+t) f(x))Q (t)dt| + |(f(x+t) f(x))Q (t)dt|


n n
1
1

+ |(f(x+t) f(x))Q (t)dt| n



- -

(|f(x+t)| + |f(x)|) Q (t)dt+|(f(x+t) f(x))(M+M) Q (t)dt Q (t)dt


n n n
1 1
- 1

< (M+M) Q (t)dt+(M+M)Q (t)dt(by(7))


n n
1
- 1

2M (
n 1 dt+ 2
2
)
Qn(t)dt+2M n(1 )dt(by(4))
2

2 n 2 n
< 2Mn 1 ( )
+2 (1)+2Mn(1 ) (1 )(by(2))
2 n
< 4Mn 1 ( ) + 2.


< 2 + 2 , for large enough n
=
Therefore lim Pn(x) = f(x) uniformly on[a,b].
n

Double click this page to view clearly


Cor: For every interval [a,a] there

is a sequence of real polynomials P n

such that P n (0) = 0 and such that

lim Pn(x) = |x| uniformly on [a,a]


n

Proof:

By the above theorem, there exists

a sequence {P n *} of real polynomials

which converges uniformly to |x| on

[a,a]. In particular, P n *(0) 0 as n

. The polynomials P n (x) = P n *(x)

P n *(0) for n = 1,2,3,..have the

desired properties.

Defintion:

A family a of complex functions

defined on a set E is said to be an

algebra if (i) f+g ? (ii)fg? (iii) cf


? for all f ?, g ? and for all

complex constants c, that is, if ? is

closed under addition, multiplication,

and scalar multiplication. For the

algebra of real functions , we have to

consider (iii) for all real c.

If ? has the property that f ?

whenever f n ? ( n = 1,2,3,....) and

f n f uniformly on E, then ? is said

to be uniformly closed.

Let be the set of all functions which

are limits of uniformly convergent

sequence of members of ? . Then is

called the uniform closure of ?.

Theorem 2.7.2: Let be the uniform

closure of an algebra ? of bounded


functions. Then is a uniformly

closed algebra.

Proof:

If f and g , there exist

uniformly convergent sequences

{f n },{g n }such that f n f , g n g

and f n ? ,g n ? . Since the functions

are bounded, we have f n + g n f +

g, f n g n fg, cf n cf, where c is any

constant, the convergence is uniform

in each case.

Hence f +g , fg and cf

Therefore is an algebra.

SinceB is the closure of ? , is closed

(uniformly).
Definition: Let ? be a family of

functions on a set E. Then ? is said

to separate points on E if to every

pair of distinct point x 1 ,x 2 E there

corresponds a function f ? , such

that f(x 1 ) f(x 2 ).

If to each xE there corresponds a

function g? , such that g(x) 0, we

say that ? Vanishes at no point of E.

Theorem 2.7.3:

Suppose ? , is an algebra of functions


on a set E, ? , separates points on

E, and ? ,vanishes at no point of E.

Suppose x 1 ,x 2 are distinct points of

E, and c 1 ,c 2 are constants (real if ?,

is a real algebra). Then ? , contains a


function f such that f(x 1 ) = c 1 , f(x 2 )

= c2.

Proof:

Since ? , separates points on E and ?

, vanishes at no point of E, we have

g(x 1 ) g(x 2 ), h(x 1 ) 0, k(x 2 ) 0,

where g,h,k ? ,.

Put u = gk g(x 1 )k, v = gh g(x 2 )h.

Since g,h,k ? , and ? ,is an algebra,

u,v ? , .

Also u(x 1 ) = g(x 1 )k g(x 1 )k = 0 and

v(x 2 )= g(x 2 )h g(x 2 )h = 0,

u(x 2 ) = g(x 2 )k g(x 1 )k 0 & v(x 1 )

= g(x 1 )h g(x 2 )h 0.

c1v c2u
Let f= + .
v(x1) u(x2)
c1v(x1) c2u(x1)
Then f(x1) = + = c1 + 0 = c1 and
v(x1) u(x2)

c1v(x2) c2u(x2)
f(x2) = + = 0 + c2 = c2
v(x1) u(x2)

STONE'S GENERALIZATION OF THE


WEIERSTRASS THEOREM

Theorem 2.7.4:

Let ? be an algebra of real continuous

functions on a compact set K. If ?

separates points on K and if ?

vanishes at no point of K, then the

uniform closure of ? consists of all

real continuous functions on K.

We shall divide the proof into four

steps.

STEP 1: If f then |f|


Proof

Let a=sup |f(x)|, ( xK ). ------------(1)

Let > 0 be given.

By the corollary to the Stone-


Weierstrass theorem, there exist real

numbers c 1 ,c 2 ,....,c n such that

| |
n

ciy |y| < ,


i y [-a,a]. -------(2)
i=1

Since is an algebra and f c i


i
f for i = 1,2,... ,n.

Hence the function g= cf i


i
is a
i=1

member of .

By (1) and (2), we have |g(x) -


|f(x)|| < (xK)
Since is uniformly closed and g(x)

, |f| .

STEP 2: If f and g , then

max(f,g) and min(f,g) .

By max(f,g) we mean the function h

defined by

{
f(x) if f(x) g(x),
h(x) =
g(x) if f(x)<g(x),

{
f(x) if f(x) < g(x),
and min(f,g) = .
g(x) if f(x) g(x),

Proof:

Consider the identities max


f+g |f-g|
max(f,g) = 2 + 2

f+g |f-g|
and min(f,g) = 2 2 .
Since is an algebra and f and

g , we have

f+g |f-g|
f+g,f-g B. Also 2 , 2 B

Therefore max(f,g) and min(f,g)

By iteration, the result can be

extended to any finite set of

functions, (i.e.) if f 1 ,f 2 ,.. .,f n

, then max(f 1 ,f 2 ,...,f n ) and

min(f 1 ,f 2 ,.. .,f n )

STEP 3: Given a real function f,

continuous on K, a point xK, and

> 0, there exists a function g x

such that g x (x) = f(x) and g x (t) > f(t)

. (tK)
Proof:

By hypothesis ? , and ? satisfies

the hypothesis of theorem 2.7.3,

also satisfies the hypothesis of

theorem 2.7.3.

Hence, for every yK, we can find a

function h y such that

h y (x)=f(x),h y =f(y) (refer theorem

2.7.3)-------(3)

By the continuity of h y there exists

an open set J y , containing y, such

that

( )
hy(t) > f(t) . t Jy -------------(4)

Since K is compact, there is finite set

of points y 1 ,y 2 ,...,y n such that


K Jy1 Jy2 ... Jyn. -------------(5)

Put g x = max(h y1 ,h y2 ,...,h yn ).

By step 2, g.

By (3), (4) and (5), we have g x (x) =

f(x) and g x (t) > f(t) .

STEP 4: Given a real function f,

continuous on K, and > 0, there

exists a function h such that |h(x)

f(x)| < . (xK)

Proof:

Let us consider the functions g x , for

eaph xK, constructed in Step 3.

By the continuity of g x , there exist

open sets V x containing x, such that


( )
gx(t) < f(t) + . t Vx . -----------(6)

Since K is compact, there exists a

finite set of points x 1 ,X 2 ,...,x m such

that

K Vx1 Vx2 ... Vxm. -----------(7)

By Step 2, h .

By Step 3, h(t) > f(t) . (tK).

(6) and (7) implies that h(t) < f(t) +


. (t K).

(i.e.) f(t) < h(t) < f(t) + (t K).

(i.e.) < h(t) f(t) < (t K).

(i.e.) |h(t) f(t)| < (t K).


Theorem 2.7.5:

Suppose ? is a self-ad joint algebra

of complex continuous functions on

a compact set K, ? separates points

on K and ? vanishes at no point of

K. Then the uniform closure B of ?

consists of all complex continuous

functions on K. In other words ? is

dense in C(X).

Proof:

Let ? r be the set of all real functions

on K which belong to ? If f ? and f =



u + iv, with u,v real, then 2u = f + f ,

and since ? is a self-ad joint, we see

that u ? r .

If x 1 x 2 , there exists f ? such

that f(x 1 ) = 1, f(x 2 ) = 0.


Hence 0 = u(x 2 ) u(x 1 ) = 1.

Therefore ? r separates points on K.

If x K, then g(x) 0 for some g

? , and there is a complex number

such that g(x) > 0.

If f = g, f = u+iv, it follows that u(x)

> 0.

Hence ? R vanishes at no point of K.

Therefore ? R satisfies the hypothesis

of theorem 2.7.4.

It follows that every real continuous

function on K lies in the uniform

closure of ? R , and hence lies in . If

f is a complex continuous function on

K, f = u + iv, then u , v .
Hence f .

CYP QUESTIONS:

1. Prove that the set C (X) of all

complex valued, continuous,

bounded functions with domain

X, with d(f,g) = ||f g|| is a

metric space.

2. Distinguish between uniformly

convergent and point wise

convergent.
UNIT-3

Unit Structure

Section 3.1: Power series

Section 3.2: The exponential and

Logarithmic functions

Section 3.3: The trigonometric

Functions

Section 3.4: The algebraic

completeness of the Complex

Field

Section 3.5: Fourier series

Section 3.6: The Gamma

function
Introduction

In this unit we shall derive some

properties of functions which are

represented by power series. We also

discuss the concept of the

Exponential, Logarithmic,

Trigonometric functions , Gamma

function and Fourier series.

SECTION-3.1 POWER SERIES

We have already discussed about the

power series in Analysis I in the first

year. The form of the power series


is f(x) = cnx
n
or more generally
n=0


n
f(x) = cn(x-a)
n=0
These are called analytic functions.

We shall discuss the power series

only for real values of x. Instead of

circles of convergence we shall

encounter intervals of convergence.

The series f(x) = n


cnx converges
n=0

for all x in ( R ,R), for some R.


n
If the series f(x) = cn(x-a)
n=0

converges for |x a | < R, then f is

said to be expanded in a power series

about the point x = a.

Theorem 3.1.1:

Suppose the series cnx


n

n=0

converges for |x| < R, and define


f(x) = cnx
n
(|x| < R) . Then the
n=0

series cnx
n
converges uniformly
n=0

on [ R + ,R s], no matter which

> 0 is chosen. The function f is

continuous and differentiable in ( R

,R), and

f '(x) = n-1
ncnx (|x| < R)
n=0

Proof:

Let > 0 be given.

n
For |x| R , we have |c n x |
n
|c n (R ) |.


n
Since cn(R-) converges
n=0

absolutely ( every power series

converges absolutely in the interior

of its interval of convergence , by the

root test)

the series c x n
n
converges
n=0

uniformly on [ R + , R ].(by

theorem 2.2.3)

Since lim n = 1 we have


n


limsup n|cn| = limsup |cn|
n n

n n

Therefore the series f(x) = cnx


n

n=0

and the series f '(x) = ncnx


n-1
have
n=1

the same interval of convergence.

Since f '(x) = ncnx


n-1
is a power
n=1

series, it converges uniformly in [

R + ,R ], for every > 0 we

can apply theorem 2.5.1( for series

instead of sequence), (i.e.)


f '(x) = nc x n
n-1
holds for |x|<R
n=1

holds if |x| R .

But, given any x such that |x| < R,

we can find an > 0 such that |x|<R

This shows that f '(x) = nc x n


n-1

n=1
holds for |x| < R.

Cor: Under the hypothesis of the


above theorem, f has derivatives of

all orders in ( R ,R), which are given

by

f( )(x) =
k
n(n-1)....(n-k+1)cnx
n-k

n=k

(k)
In particular, f (0) = k!c k , (k =

0,1,2,....).

Proof:

By the above theorem from


f(x) = n
cnx we get
n=0

f '(x) = ncnx
n-1

n=1
Apply theorem 3.1.1, to f , we get

Successively apply theorem 3.1.1 to f


(3)
,f ,...., we get


(k)
(x) = n(n-1)....(n-k+1)cnx , -----------(1)
n-k
f
n=1

(k)
Putting x = 0 in (1), we get f (0)=

k!c k , (k = 0,1,2,....).

Abel's Theorem

Theorem 3.1,2:

Suppose c n converges. Put


f(k) = c x ( 1 < x<R)


n
n

n=1

Then lim f(x) =


x1
c n
n=1

Proof:

Let s n = c 0 + c 1 + ....+ c n , s 1 = 0.
m m

Then c x n
n
= (s n sn-1)x
n

n=0 n=0

= (s0 s 1)x + (s1 s0)x + (s2 s1)x + ... + (sm sm-1)x


0 1 2 m

1 1 2 2 m-1 m
= s01 + s1x s0x + s2x s1x ..... sm-1x + smx
1 2 m-1 m
= (1 x)s0 + (1 x)s1x + (1 x)s2x ....(1 x)sm-1x + smx
m-1

= (1-x) s x n
n
+ smx
m

n=0

For |x| < 1, let m ,we get c x n


n
= (1 x) s x n
n

n=0 n=0

(i.e.)f(x) = (1 x) snx
n

n=0

Since c n converges to s (say).

Then its partial sum sequence {s n }

converges to s.

(i.e.) lim sn = s.
n

Let > 0 be given.

Choose N so that n>N |s-sn| < 2 . ----------(1)

By the geometric series test,we have x n


= 1 if |x|<1.
n=0

Double click this page to view clearly


(i.e)(1 x) x = 1 if |x| < 1. ------------(2).


n

n=0

Now to prove that


x1
lim f(x) = c n
=s
n=0

| || |

|f(x)-s| = (1-x) snx (1 x) snx s(1 x) snx (by(2))


n n n
s =
n=0 n=0 n=0

| |

= (1-x) (s -s)x n
n

n=0

(1 x) |s -s||x | ( |x+y| |x| + |y|)


n
n

n=0

|s -s||x| |s -s||x|
n n
(1 x) n + (1 x) n
n=0 n=N+1
n

|s -s||x| (1 x) |x| (by(1))


n n
(1 x) n + 2
n=0 n=N+1
N

|s -s||x|
n
< (1 x) n + 2. ---------------(3)
n=0

Choose > 0 such that 1-x <

Then (3) becomes

|f(x) s| < 2 + 2 =

(i.e.)|f(x) s| < if x>1-.


(i.e) lim f(x) = s = cn


x1
n=0

Double click this page to view clearly


Note:If a n , b n , c n , converge

to A,B,C , and if c n = a 0 b n + a 1 b n1

+.... + a n b 0 , then C = AB.

Theorem 3.1.3:

Given a double sequence {a ij }, i =

1,2,3,..., j = 1,2,3,...,

suppose that |aij| = bi (i=1,2,3,....) and biconverges.Then aij = a . ij


j=1 i=1 j=1 j=1 i=1

Let E be a countable set, consisting

of the points x 0 ,x 1 ,x 2 ,...,x n , and

suppose x n x 0 as . Define

Proof:

Let |a | = b ij i (i=1,2,3,....) -----------------(1)


j=1

fi(x0) = a (i=1,2,3,...)
ij ----------(2)
j=1
n

fi(xn) = a (i,n=1,2,3,...)
ij ----------(3)
j=1

g(x) = a f (x)( xE )
ij i ----------(4)
i=1

Double click this page to view clearly


Now, (2) and (3), together with (1),

we get

fi(n) = a ij aij = fi(x0) as n


j=1 j=1

(i.e.) x n x 0 f i (x n ) f i (x 0 ) as n

This shows that each f i is continuous

at x 0 .

Now |fi(x)| = |a | = b ij i for x E &i=1,2,3,....


j=1

g(x) = f (x) converges uniformly.


i
i=1

By theorem 2.3.1, g is continuous at

x0.

Double click this page to view clearly


Therefore a ij = f (x ) (by(2))
i 0
i=1 j=1 i=1

=g(x0) (by(4))

= lim g(xn)
n

= lim
n i=1
f (x )(by(4))
i n

= lim
n
(a )(by(3))
i=1 j=1
ij

= lim
n
(a )(by(3))
j=1 i=1
ij

= lim
n
(a ) = a
j=1 i=1
ij ij
j=1 i=1

a ij = a ij
i=1 j=1 j=1 i=1

TAYLOR'S THEOREM

Theorem 3.1.4:

Suppose f(x) = cnx


n
, the series
n=0

converging in |x| < R. If R< a < R,

then f can be expanded in a power

series about the point x = a which

Double click this page to view clearly


converges in | x a | < R |a|, and

f( )(a)
n


n
f(x) =
n!
( x-a) (|x-a| < R-|a|)
n=0

Proof:

We have f(x) = c x n
n

n=0


n
= cn(x-a+a)
n=0


n
= cn((x-a)+a)
n=0

( )
n

( )
n n-m m
= cn a (x-a)
m
n=0 m=0

Therefore f(x) can be extended in the

form of power series about the point

x = a.

|() n n-m
| |c |(|x-a| + |a|)
m n
But cn a (x-a) = n
m
n=0 m=0 n=0

which converges if |x a| + |a| < R.

(i.e.) if |x a| < R |a|


We know that, by corollary to the

theorem 3.1.1,

f( )(0) = k!ck.
k
(k=0,1,2,....).
f( )(0)
k

(i.e.)ck = k! (k=0,1,2,...)

f( )(a)
n


n
f(x) =
n!
( x-a) (|x-a| < R-|a|)
n=0

Theorem 3.1.5:

n n
Suppose the series a n x and b n x

converge in the segment S = (R,R).

Let E be the set of all xS at which

n
anx = n
bnx
n=0 n=0

If E has a limit point in S, then a n =

b n for n = 0,1,2,3, ....


Hence anx =
n
n
bnx holds for all
n=0 n=0

xS.

Proof:

Put cn = an bn and

f(x) = n
cnx ( xS )
n=0

= (a n bn)x = 0 for x S.(by hypothesis)


n

n=0

Let A be the set of all limit points of

E in S. Let B be the set of all other

points of S.

By the definition of limit point, B is

open.

Let x 0 A.
By the above theorem,

(|x-x0| < R-|x0|)


n
f(x) = dn(x-x0)
n=0

Now to prove that d n = 0 for all n.

Suppose there exist a smallest non-

negative integer k such that d k 0.

( )
k
f(x) = (x-x0) g(x) |x-x0| < R-|x0| ,


m
where g(x) = dn+k(x-x0)
m=0

Since g is continuous at x 0 , and g(x 0 )

= d k 0, there exists a > 0 such

that g(x 0 ) = d k 0 if |x x 0 | < .

k
Therefore f(x) = (x x 0 ) g(x) 0

if 0 < |x x 0 | < , which is a


contradiction to the fact that x 0 is a

limit point of E.

Therefore d n = 0 for all n.


n
So f(x) = dn(x-x0) = 0 if |x-x0| < R-|x0|
n=0

(i.e.) in a neighborhood of x 0 .

Therefore A is open.

(i.e.) A and B are disjoint open sets.

(i.e.) A and B are separated.

Since S = A B and S is connected,

one of A and B must be empty.


By hypothesis A is not empty.

Hence B is empty and A = S.

Since f is continuous in S, A E. Thus

E = S and
f( )(0)
n

cn = n! = 0 (n=0,1,2,....)

an = bn for n=0,1,2,3,....

anx =
n
n
bnx holds for all x S
n=0 n=0

CYP QUESTIONS:

{ e
1 / x2 ( x0 )
1. Define f(x) = .Prove that f has derivatives of all
0 (x=0)

(n)
orders at x = 0 and that f (0) =

0 for n = 1,2,3,...

SECTION-3.2 THE
EXPONENTIAL AND LOGARITHMIC
FUNCTIONS

Definition:

n
z
E (z ) = n!
n=0
Note 1: By the ratio test, E(z)

converges for every complex z.

Note 2: (Addition formula)



n m
z w
E (z )E (w ) = n! m!
n=0 m=0
n
k n-k
z w
= k!(n-k) !
n=0 k=0
n n

( )
1 n k n-k (z+w)
=
n! k
z w = n!
n=0 k=0 n=0

= E(z+w), where z and w are complex numbers.

Note 3: E(z)E(z) = E(z z) = E(0)

= 1, where,z is a complex number.

E(z) 0 for all z.

By the definition, E(x) > 0 if x > 0


and E(x) > 0 for all real x.

Again by the definition, E(x) + as

x + and E(x) 0 as x along

the real axis.

Double click this page to view clearly


Also 0 < x < y implies E(x) < E(y)

and E(y) < E(x).

Hence E is strictly increasing on the

whole real axis.

E(z+h) E(z) E(h) 1


lim h = E(z)lim h = E(z).1 = E(z)
h=0 h=0

By iteration of the addition formula

gives

E(z 1 + z 2 + ...+ z n ) = E(z 1 )E(z 2 ). .

.E(z n ).

If z 1 = z 2 = ...= z n = 1,then E(n) =


n
e ,n= 1,2,3,...(E(1) = e)

If p = n/m, where n and m are

positive integers, then


m n
[E(p)] = E(mp) = E(n) = e , so that
p p
E(p) = e and E(p) = e (p>0, p

rational).
Theorem 3.2.1:

x
Let e be defined on

n
x
x
e = E(x) = n!
n=0

x
a. e is continuous and

differentiable for all x;

x x
b. (e ) = e ;
x
c. e is a strictly increasing function
x
of x, and e > 0;

x+y x y
d. (d) e = e e ;

x x
e. e + as x+ and e 0 as

x ;
n x
f. lim x e = 0 for every n.
x +

Proof:

For the proof of (a) to (e), refer Note

1,2,3.

n
x
(f)By definition e =
x
n!
n=0
2 n n+1
x! x x x
=1+ 1 ! + 2! + .... + n! + (n+1) ! + .....
n+1
x
> (n+1) ! for x>0

e (n+1)!
e (n+1) !
0 as x +;
n
x x x x -x n
(i.e.)e > (n+1) !
< n
x x
x < x

n -x
(i.e.) lim x e = 0 for every n.
x +

Note:Since E is strictly increasing


1
and differentiable on R , it has an

inverse function L, which is also

strictly increasing and differentiable


1
and whose domain is E(R ), that is,

the set of all positive numbers, L is

defined by E(L(y)) = y (y > 0) or

L(E(x)) = x (x real).

CYP QUESTIONS:

1. Find the following limits.


1/x
e-(1 + x)
i. lim x
x0
ii.
n
lim

n
log n [ 1/n
n ]
1

SECTION-3.3 THE
TRIGONOMETRIC FUNCTIONS

Let us define

[E(ix) + E(-ix)] and S(x) = 2i [E(ix) E(-ix)]


1 1
C(x) = 2

()

Since E z = E(z), C(x) and S(x) are

real for real x.

Also E(ix) = C(x) + iS(X) (i.e.) C(x)

and S(x) are real and imaginary parts

of E(ix),respectively, if x is real.
2
|E(ix)| = E(ix)E(ix) = E(ix)E(-ix) = 1, so that |E(ix)| = 1(x real)
, so that |E(ix)| = 1 (x real).

Also C(0) = 1 and S(0) = 0 , and

C'(x) = S(x), S'(x) = C(x).

Double click this page to view clearly


Now to show that there exist a

positive real number x such that

C(x)=0. Suppose C(x) 0 for all x.

Since C(0) = 1, we have C(x) > 0 for

all x > 0.

Since S(0) = 0 and S'(x) = C(x), we

have S'(x) > 0 for all x > 0.

S(x) is strictly increasing function.

If 0 < x < y then S(x) < S(y).

y y

(i.e.)S(x)(y-x) < S(t)dt < C '(t)dt = -(C(t))x = [C(y) C(x)]


y

x x

= [C(x) C(y)]

2 2 2
Since 1 = |E(ix)| = C (x) + S (x),
2
C (x) 1.

Double click this page to view clearly


C(x) and C(x) 1.

[C(x) C(y)] 2.

(i.e.)S(x) (y x) 2.

Since S(x) > 0 this inequality cannot

be true for large values of y , we get

a contradiction.

a positive number x such that

C(x) = 0.

Let x0 be the smallest positive

integer such that C(x 0 ) = 0.

Define the number by = 2x 0 .

Then C(/2) = C(x 0 ) = 0

Since E(ix) = C(x) + iS(X) & |E(ix)|


= 1, we have S(/2) = 1
Since C(x) > 0 in (0, /2), S is an

increasing function in (0, /2).

Therefore S(/2) = 1. Thus ( )=i


i
2

E(i)=E ( i2 + i2 )=E( i2 ).E( i2 )=i.i=-1


E(2i)=E(i+i)=E(i).E(i) = ( 1)( 1) = 1.
Also E(z+2i)=E(z).E(2i)=E(z)

Theorem 3.3.1:

a. The function E is periodic, with

period 2i,

b. The functions C and S are

periodic, with period 2,

c. If 0 < t < 2, then E(it) 1,

d. If z is a complex number with |z|

= 1, there is a unique t in [0, 2)

such that E(it) = z.


Proof:

a. Since E(z+2i) = E(z), E is

periodic, with period 2i.

b. C(x+2) =
1
2 [E(i(x+2)) + E(-i(x+2))]
= [E(ix+2i) + E(-ix-i2))]
1
2

= 2 [E(ix) + E(-ix)](Since E(2i) = 1)


1

=C(x)

Therefore C(x) is periodic with

period 2.

Similarly S(x) is periodic with

period 2.

c. Suppose 0 < t < /2 and E(it) =


x + iy, where x & y are real.

0 < x < 1 & 0 < y < 1.

2 2 2
1 = |E(it)| = x + y .
4 4 4
E(4it) = (E(it)) = (x + iy) = x
2 2 4 2 2
6 y + y + 4ixy(x y )
2 2
If E(4it) is real, then x y = 0
2 2
, that is x = y .
2 2 2 2
Since x + y = 1,2x = 1 x =
2
and y = .
2 2 2
E(4it) = (1/2) 6(l/2) (l/2)
2
+ (1/2) + 4ixy(0)
= 1/4 6/4 + 1/4 = 4/4 =

1.

E(it) 1.

d. Choose z so that |z| = 1.

Write z = x + iy, with x and y


real.

Suppose first that x 0 and y

0.

On [0,/2], C decreases from 1

to 0.

Hence C(t) = x for some t[0, /

2].
2 2
Since C + S = 1 and S 0 on

[0,/2], it follows that z = E(it).

If x < 0 and y 0, the preceding

conditions are satisfied by iz .

Hence iz = E(it) for some t[0,

/2], and since i = E(i/2), we

obtain z = E(i(t+/2)).

Finally if y < 0, the preceding

two cases show that z = E(it)

for some t(0, ). Hence z =

E(it) = E(i(t+)).

Suppose 0 t 1 < t 2 < 2,


1
E(it 2 )[E(it 1 ) = E(i(t 2 t 1 ))

1( by (c))

Therefore there is a unique t in

[0,2) such that E(it) = z.

CYP QUESTIONS:
2 sinx
1. If 0<x< 2 , prove that < x <1
SECTION-3.4 THE ALGEBRAIC
COMPLETENESS OF THE COMPLEX
FIELD

Theorem 3.4.1:

Suppose a 0 ,a 1 ,...,a n are complex


n

numbers, n 1, a n 0,P(z) = akz


k

. Then P(z) = 0 for some complex

number z.

Proof:

Without loss of generality we assume

that a n = 1.

Put = inf |P(z)| (z complex) .

If |z| = R, then |P(z)| = | a 0 + a 1 z +


n n
... + a n z | = | a 0 + a 1 z + ... + z |
n n1
|z | |a n 1 z | ..... |a 0 |
n n1
R |a n1 |R .... |a 0 |

n 1 n
= R [1 |a n1 |R ..... |a 0 |R ]

as R

Hence there exist R 0 such that |P(z)|

> if |z| > R 0 .

Since |P| is continuous on the closed

disc with centre at 0 and radius R 0 ,

|P(z 0 )| = for some z 0 .

Claim: = 0.

Suppose 0.

P(z+z0) P(z0)
Put Q(z) = .then Q(0) = = 1.
P(z0) P(z0)

| |
Since P(z0) = =inf |P(z)|, P(z+z0) P(z0)

P(z+z0)
1.(i.e) Q(z) 1 for all z.
p(z0)
there is a smallest integer k, 1 k
k
n, such that Q(z) = 1 + b k z + a 1 z
n
+.... + b n z , b k 0.

By theorem 3.3.1(d), there is a real


ik
such that e b k = |b k |.

k ik
If r > 0 and |b k | < 1, |1 + b k r e |
k
= 1 r |b k | so that

i k
|Q(re )| 1 r {|b k | r|b k+1 |
nk
.... r |b n |}

For sufficiently small r, the

expression in braces is positive.

Hence

i
|Q(re )| < 1, which is a contradiction

to Q(z) 1 for all z.


= 0, that is P(z 0 ) = 0.

SECTION-3.5 FOURIER SERIES

Definition:

A trigonometric polynomial is a finite

sum of the form


N

f(x) = a0 + (a cos nx+b


n n sin nx)(x real)
n=1

where a 0 ,a 1 ,.. .,a N ,b 0 ,b 1 ,.. .,b N are

complex numbers,Since

[E(ix) + E(-ix)] and S(x) = 2! [E(ix) E(-ix)], f(x)


1 1
C(x) = 2

can also be written in the form


N

f(x) = c e n
inx
(x real)
N

Note 1: Every trigonometric

polynomial is periodic with period

2.Note 2: If n is a nonzero integer,


inx inx
e is the derivative of e /in,

which is also has period 2. Hence

{

1 if n=0
1
2 e inx
dx =
0 if n=1,2,...
-

If we multiply f(x) = inx


cne by e
imx
,
-N

where m is an integer, if we integrate

the product, we get


cm= 2
1
f(x) e -imx
dx for |m| N.
-

If |m| > N, the above integral is zero.

Note 3: The trigonometric polynomial


N

f(x) = cne
inx
is real iff c-n = cn for n
N

= 0,1,...,N.
Note 4: We define a trigonometric

series to be a series of the form


inx
cne (x real) the nth partial sum

N

of this series is defined to be c e n


inx

Note 5: If f is an integrable function

on [,], the numbers c m for all

integers m are called the Fourier

coefficients of f, and the series


inx
cne formed with these

coefficients is called the Fourier

series of f.
Definition:

Let { n} (n = 1,2,3,...) be a

sequence of complex functions on

[a,b], such that




n(x)m(x) dx=0( nm ) (n m).

Then { n } is said to be an

orthogonal system of functions on


b

| |
2
[a,b]. If, in addition n(x) dx = 1,
a

, for all n, { n } is said to be

orthonormal.

Example:
1
inx
The functions (2) 2
e form an

orthonormal system on [,].

Note: If { n } is orthonormal on

[a,b] and if
b

cn = f(t)n(t) dt (n=1,2,...) we call c n
a

the nth Fourier coefficients of f

relative to { n }.

We write f(x) c (x) and call this


n n
1

series the Fourier series of f.

Theorem3.5.1:

Let{ n } be orthonormal on [a,b].

Let sn(x) = c mm (x) be the nth


m=1

partial sum of the Fourier series of


n

f, and suppose tn(x) = mm (x)


m=1
b b


2 2
.Then |f-sn| dx |f-tn| dx and
a a
equality holds iff m =c m ,(m =

1,2,...,n).
Proof:

Let denote the integral over [a,b],

the sum from 1 to n.

Then

ftn =
f

mm=

m

fm =

mcm

(by the definition of cm).


n n n n

|t |


2
Now n = tntn = mm mm = mm mm
m=1 m=1 m=1 m=1

| | (since { } is orthonormal)
2
= m n

b b b

|f-t | dx =(f-t )( ) (f f ft )
2
n n f -tn dx= n tn f + tntn dx
a a a
b b b b

= f f dx- ftndx- tn f dx+ t t dx n n
a a a a
b b b b

|f| dx- |t | dx
2 2
= ftndx- tn f dx+ n
a a a a
b

= |f| dx- c
a
2
mm
c
m m m m

|f| dx- |cm|


2 2 2
= + |m cm| ----------(1)
a

Double click this page to view clearly


which is evidently minimized iff

m cm

b b

|f-sn| |f-tn|
2 2
dx dx
a a
b b

|f-t | dx=|f| dx- |c


2 2 2
Put in m = cm(1), we get n |
m
a a
b b

|f-tn| |f| dx- |cm| 0


2 2 2
Since dx 0,
a a
b

|f| dx |c
2 2
m |
a
n b

|s (x)| dx = |c |f(x)| dx
2 2 2
n m |
m=1 a

Theorem3.5.2:

(Bessel's inequality) If { n } is

orthonormal on

[a,b] and if f(x) cnn(x), then |cn| |f(x)| dx


2 2

n=1 n=1 a

.In particular lim cn = 0.


n

Double click this page to view clearly


Proof:

From the above theorem, we have

n b

|sn(x)| |c |f(x)| dx.


2 2 2
dx = m|
m=1 a

|cn| |f(x)|
2 2
Letting n ,we get dx
n=1 a

Also lim cn = 0.( ? )


n

Trigonometric series: We shall

consider functions f that have period

2 and that are Riemann-integrable


on [ ,]. The Fourier series of f

is given by

f(x) = c e n
inx
where cn =
1
2 f(x)e -inx
dx
-

, the Nth partial sum of the Fourier

series of f is given by

Double click this page to view clearly


N

SN(x) = SN(f;x) = c e n
inx

-N

|SN(x)| |cn| |f(x)|


1 2 2 1 2
2 dx = 2 dx
- n=-N -

The Dirichlet kernel DN(x) is defined by

e inx sin(N + 1/2)x


DN(X) = = sin(x / 2)
n = N
N N
1
Since cn =
1
2 f(x)e -inx
dx,SN(f;x) = c e n
inx
= f(t)e
2
-int inx
dte
- N -N -
N N
1
= f(t)e 2
-in(x-t) 1
dt= 2 f(t) e -in(x-t) 1
dt= 2 f(t)D (x-t)dt N
N - - -N -

= 2
1
f(x-t)D (t)dt N
-

Theorem 3.5.3:

If, for some x, there are constants

> 0 and M < such that


|f(x+t) f(x)| M|t| for all t (-,), then lim SN(f;x) = f(x)
N

Double click this page to view clearly


Proof:
f(x-t) f(x)
Define g(t) = sin(t/2)
for 0<|t| < , and put g(0) = 0
N
1
Now
1
2 DN(x)dx=
2 e inx
dx
- - n=-N

(by the definition of Dirichlet kernel)


N N

=
1
2 e inx
dx= e inx
dx
n=-N - n=-N

( { )

1 if n=0
=1
1
2 e inx
dx=
0 n=1, 2, ...
-

SN(f;x) f(x) =
1
2 f(x-t)D (t)dt-f(x) = f(x-t)D (t)dt-f(x).1
N
1
2 N
- -

=
1
2 f(x-t)D (t)dt-f(x) D (t)dt
N
1
. 2 N
-

1
=
1
2 (f(x-t)-f(x))DN(t)dt= 2 g(t)sin(t/2)D (t)dt N
- -

g(t) sin(t/2) g(t) sin(N+1/2)dt


1 sin(N+1/2)t 1
= 2 sin(t/2)
dt= 2
- -

1
= 2 g(t) [sin Nt cos t/2+cos Nt sin t/2]dt
-

1
= 2 [g(t)cos t/2]sin Nt 1
dt+ 2 [g(t)sin t/2]cos Nt dt
- -

By |f(x+t) f(x)| M|t| for all

t(,) and the definition of g(t),

g(t)cos(t/2) and g(t)sin(t/2) are

bounded.

Double click this page to view clearly


The last two integrals tend to 0 as N

(by theorem 3.5.2)

lim SN(f;x) = f(x)


N

Theorem 3.5.4:

If f is continuous (with period 2) and

if > 0, then there is a trigonometric

polynomial P such that |P(x) f(x)|

< for all real x.

Proof:

If we identify x and x + 2, we may

regard the 2-periodic functions on


1
R as functions on the unit circle T,
ix
by means of the mapping x e . The

trigonometric polynomials , i.e., the


N

functions of the form f(x) = cne


inx
,
-N
form a self-adjoint algebra ? , which

separates point on T, and which

vanishes at no point of T. Since T is

compact, ? is dense in ?(T).

Theorem 3.5.5:

(Parseval's theorem) Suppose f

and g are Riemann-integrable

functions with period 2, and


f(x) c e n
inx
, g(x) e n
inx



|f(x)-S (f;x)| c ,
1 2 1
Then lim 2 N dx=0, 2 f(x) g(x) dx= n n
N - -

|f(x)| dx= |c |
1 2 2
2 n
-

Proof:

Let us use the notation

{ }
1
2


2
|h(x)| dx
2 1
h = 2
-

Double click this page to view clearly


Let > 0 be given.

Since f and f() = f(), there

exists a continuous 2-periodic

function h with ||f h|| 2 < .

By the above theorem, there is a

trigonometric polynomial P such that

|h(x) P(x)| < for all x.

{ } { } { }
1 1 1
2 2 2

|h(x)-P(x)| dx dx dx
2 2
1 1 2
h-P 2 = 2 < 2 = 2
- - -

{ }
2
2
= 2 2 =

If P has degree N 0 , theorem 3.5.1

shows that ||h S n (h)|| 2 < ||h

P|| 2 <, for all N N 0 .

Double click this page to view clearly


SN(h) SN(f) 2 = SN(h-f)2 2 h-f 2 < .(by theorem 3.5.1)

Now f-SN(f) 2 = f-h+h-SN(h) + SN(h) SN(f) 2

f-h 2 + h-SN(h) 2 + SN(h) SN(f) 2

<++ = 3.
lim f-SN(f) 2 = 0
N

{ }
1
2
2
|f(x)-SN(f;x)|
1
(i.e) lim 2 dx =0
N -

{ }

2
|f(x)-SN(f;x)|
1
(i.e) lim 2 dx = 0
N -

N
inx inx
Next, 2
1
1
SN(f)gdx= 2 cne g(x)dx= cn 2
1
e g(x) dx
- - -N -
N

= c n n ---------------(1)
n=-N

By Schwarz inequality,we have

| | { | |}
1

||
2
2 2

fg SN(f)g |f-SN(f)| g |f-SN(f)| g 0 as N

(i.e.)
|
fg

SN(f)g
| 0 as N

lim
N -


SN(f)g =

fg



(i.e.) lim
N
1
2 SN(f)g =
1
2 fg
- -

Double click this page to view clearly


N

By(1), lim
N n=-N
c n n =
1
2 fg
-


(i.e.) 2 fg = cnn
1

- n=-


(i.e.) 2
1
f(x)g(x) dx = cnn
- -

Put f = g in the above equation, we

get

| |
2 2
|cn|
1
2 f(x) dx =
- n=-

CYP QUESTIONS:

1. Suppose 0 < < , f(x) = 1 if |x|


, f(x) = 0 if < |x| , and

f(x+2) = f(x) for all x.

a. Compute the Fourier

coefficients of f.
2
sin (n) -
b. Conclude that n
= 2

Double click this page to view clearly


SECTION 3.6 THE GAMMA
FUNCTION

Definition

For 0<x<,(x) t
x-1 -t
e dt
0

The integral converges for x (0, )

Theorem 3.6.1:
a. The functional equation (x+1) =

x(x) holds if 0 < x <

b. (n+1) = n! for n = 1,2,....

c. log is convex on (0, ).

Proof:
a. Let 0 < x < .

(x) = t x-1 -t
e dt
0

(x+1) = t x+1-1 -t
e dt= t e x -t
dt= t d(e )
x -t

0 0 0

= ( t e )0 + xt
x -t x-1 -t
e dt=0+x t x-1 -t
e dt=x(x)
0 0

Double click this page to view clearly


By (a)(n+1) = n(n) = n(n-1)(n-1)
=n(n-1)(n-2)....(1)(1)

b.
(1) = t 1-1 -t
e dt= t e 0 -t
dt= e -t
dt=1
0 0 0

(n+1) = n(n+1)(n-2).....1 = n!

1 1
Let 1<p< and p + q = 1.

( )= t( p q ) e dt = t( ) ( 1p + 1q )e t( 1p + 1q )dt
x y x y
x y + 1 -t +
Now p + q
p q

0 0

t( ) ( qy + 1q )e pt qt dt=
t( )e pt t( y-1q )e qt dt
x 1 x-1

= p p p

0 0

{ } { }
1/p 1/q

t x-1 -t
e dt . t y-1 -t
e dt (by Holder's inequality)
0 0

c. 1/p 1/q
= {(x)} {(x)}
Taking log on both sides,we get

( px + qy ) log[{(x)} {(y)} ] = log{(x)}


1/p 1/q 1/p 1/q
log + log{(x)}

1 1
= p log(x) + q log(y)
1 1 1
Put = p .Then 1-=1- p = q

log(x+(1 )y) log (x) + (1 )log (y)

log is convex on(0, )

Theorem 3.6.2:
If f is a positive function on (0, )

such that

Double click this page to view clearly


a. f(x+1) = xf(x)

b. f(1)= 1

c. log f is convex,

then f(x) = (x).

Proof:

Since satisfies (a), (b) and (c) , it is

enough to prove that f(x) is uniquely

determined by (a), (b) and (c), for all

x > 0.

By (a), it is enough to do this for x


(0,1).

Put =logf. Then f(x+1) = xf(x)

log f(x+1) = log x + log f(x)

(x+1) = log x+(x). ----------(1)

Since f(1) = 1, (1) = log f(1) = 0.

By (c) , is convex .
Suppose 0 < x < 1, and n is a
positive integer.

Then f(n+1) = nf(n) = n(n 1 )f(n

1) = n!

(n+1) = log f(n+1) = logn! -----(2)

Consider the difference quotients of

on the intervals

[n,n+1],[n+1,n+1+x], [n+1, n+2].

(n+1) (n) (n+1+x) (n+1) (n+2) (n+1)


n+1-n n+1+x-n-1 n+2-n-1

(n+1+x) (n+1)
log n n+1+x-n-1 log(n+1)(by(1)) ----------(3)

From(1), we have (n+x+1) = log(n+x) + (n+x)


=log(n+x) + log(n+x-1) + (n+x-1)
=log(n+x) + (n+x-1) + (n+x-1)

=....=(x) + log[x(x+1)...(x+n)]

(3) logn
(x) + log[x(x+1)...(x+n)] log n!
x log(n+1)(by(2))

x log n (x) + log[x(x+1)...(x+n)] log n! x log(n+1)


Subtract xlog n,we get

0 (x) + log[x(x+1)...(x+n)] log n!-x log n x log(n+1) x log n

0 (x) + log[x(x+1)...(x+n)] log n!-x log n x (log(n+1) x log n)


x

[ ] ( n+1
n )
x
n!n
0 (x) log x(x+1)...(x+n) x log

=x log 1 + ( 1
n ) 0 as n

Double click this page to view clearly


[ ]
x
n!n
(i.e.) (x) log x(x+1)...(x+n)
0 as n

[ ]
x
n!n
(i.e.) (x) = lim log x(x+1)...(x+n)
n

(i.e.) (x) is determined uniquely

(i.e.) log f(x) is determined uniquely

f(x) is detennined uniquely

f(x) = (x).

Theorem 3.6.3:

If x > 0 and y > 0, then


1
(x)(y)
x-1 y-1
t ( 1 t) dt=
(x+y)
0

(This integral is so called beta

function B(x,y))
Proof:

By the definition of gamma function,

we have

t x-1 y-1
(x) = (1-t) dt
0
1

t x1 y-1
Now B(x, y) = (1-t) dt
0
1 1

( )
y 1

t (1-t) dt=(1-t)
11 y-1 y1 (1-t) 1
Therefore B(1,y) = dt= y = y
0
0 0

Let p,q be real numbers such that


1 1
p + q =1
1
( p + q) 1
( )
x x
x x y-1
B(x,y) = B p + q, y = t ( ) dt
1-t
0
1
( p + q) (p + q)
y( p + q ) ( p + q )

x x 1 1 1 1 1 1

= t ( )
1-t dt
0
1

( p + q) (p + q)
(1-t) ( p q ) ( p q )dt
x x 1 1 1 1 1 1
y + +
= t
0
1

( p p) (q + q)
(1-t)( p p )(1 t)( q p )dt
x 1 x 1 y 1 y 1

= t
0
1

( p ) ( q )
(1-t)( p )(1 t)( q )dt
x-1 x-1 y-1 y-1

= t t
0

Double click this page to view clearly


{( } {( }
1 1/p 1 1/q

) )
p q
t( )(1 t)( ) t( )(1 t)( )
x-1 y-1 x-1 y-1

p p
dt p p
dt
0 0

(by Holder's inequality)

{ }{ }
1 1/p 1 1/q

t t
x-1 y-1 x-1 y-1 1/p 1/q
= (1 t) dt (1 t) dt = B(x,y) , B(x,y)
0 0

(i.e.)B( p + q , y) B(x,y)
x x 1/p 1/q
.B(x,y)

Taking log on both sides , we get

log B ( px + qx , y) 1p log B(x,y) + 1q log B(x,y)


1 1

t (1-t) dt = t (1-t) dt
x+1-1 y-1 x y-1
Now B(x+1,y) =
0 0
1 1

( )
x x
t t
(1 t)x (1-t) (1 t) dt= 1 t (1-t)
x y-1 x+y-1
= dt
0 0

( )dt and
x x-1
Let u= ( 1 t t ) , then du=x( 1 t t ) 1
(1 t )
2

1 x+y
(1 t)
(1 t)
x+y-1
dv= dt,v=-
x+y
0
1 x+y

( ) ( )
x (1-t)x+y 1 (1 t) t
x-1
1
B(x+1,y) = ( t
1t x+y ) 0
+ x+y
x
1-t (1 t )
2 dt
0
1 1

(1-t) ( ) t x-1 1
(1-t)
x x+y x x+y-x+1-2 x-1
=0+ x+y 1-t 2 dt= x+y t dt
(1-t)
0 0
1

(1-t)
x y-1 x-1 x
= x+y t dt = x+y B (x,y)
0
x
(i.e.)B(x+1,y) = x+y B (x,y). ------------(1)
(x+y)
Let f(x) = (y)
.B(x,y).

Double click this page to view clearly


(x+1+y)
Then f(x+1) = (y)
.B(x+1,y)
(x+1+y)
. x+y B(x,y). (by(1))
x
= (y)

(x+y)(x+y) x x(x+y)
= (y)
. x+y B( x,y) = (y)
.B(x,y) = xf(x)
(y+1) (1+y) 1 y(y) 1
f(1) = (y)
.B(1,y). = (y) y
= (y) y =1

log f(x) = log(x+y) + log B(x,y) log(y)

(x+1+y)
Then f(x+1) = (y)
.B(x+1,y)
(x+1+y)
. x+y B(x,y). (by(1))
x
= (y)

(x+y)(x+y) x x(x+y)
= (y)
. x+y B( x,y) = (y)
.B(x,y) = xf(x)
(y+1) (1+y) 1 y(y) 1
f(1) = (y)
.B(1,y). = (y) y
= (y) y =1

log f(x) = log(x+y) + log B(x,y) log(y)

log f(x) = log (x+y) + log B(x,y)

log (y).

Since log and log B are convex, log

f is also convex.

Double click this page to view clearly


By the above theorem f(x) = (x).
(x+y)
(i.e.) (y)
.B(x,y) = (x).
(x)(y)
B(x,y) = (x+y)

(i.e.) t (1 t) dt= (x+y)


x-1 ( ) ( ) y-1 x y

Some Consequences The


2
substitution t = sin in the above

equation turns into


/2

(
x-1 y-1
) ( )
2 2 (x)(y)
2 sin 1 sin sin cos d= (x+y)
0

/2

(i.e.)2(sin )
x-1 y-1
( )
2 2 (x)(y)
1 sin sin cos d= (x+y)
0

/2
(1 / 2)(1 / 2)
(sin ) (
2 1 / 2) 1
(cos ) ( ) d
2 1/2 1
(1/2+1/2)
=2
0
/2
( )
2
(1 / 2)
(i.e.) (1)
=2 d=2(/2)=
0

( )
2
Since (1) = 1, we have (1 / 2) =

Therefore (1 / 2) = .

Double click this page to view clearly



2
Put t=s in (x) = t x-1 -t
e dt,we get
0


2 2
2x-2 s 2x-1 s
(x) = s e 2sds =2 s e ds
0 0

Put x = , we get

= ( 1/2 ) = s e
2 2
2( 1/2 ) 1 s s
e ds = ds.
0 0

( 12 ) ( x +2 1 ) (Verify).
x1
2
Also (x) =

CYP QUESTIONS:

1. Prove the Stirling's formula


(x + 1)
lim x = 1.
( x/e ) 2x
x

( 21 ) ( x +2 1 )
x1
2
2. Prove that (x) =

3. If f(x) = 0 for all x in some

segment J, then prove that lim

S N (f:x) = 0 for every x

J.(Localization theorem)

4. Prove that lim x log x = 0 for
x +

every >0.

Double click this page to view clearly


UNIT-4

Unit Structure

Section 4.1: Linear

Transformations

Section 4.2: Differentiation

Section 4.3: The contraction

principle.

Section 4.4: The inverse function

Theorem

Introduction

In this unit we shall discuss about

the set of vectors in Euclidean n-


n
space R , linear transformation,

differentiation and the contraction


principle and finally the inverse

function theorem.

SECTION-4.1 LINEAR
TRANSFORMATIONS

Definition:

n
A non-empty set X R is a vector

space if x + y X and cx X, for

every x,y X and for all scalar c.

Definition:

n
If x 1 ,x 2 ,...,x k R and c 1 ,c 2 ,...,c k are

scalars then x 1 c 1 + x 2 c 2 +... + x k c k

is called a linear combination of x 1

,x 2 ,... ,x k .
Definition:

n
Let S R .Then the set of all linear

combination of elements of S is called

a linear span of S and it is denoted

by L(S).

Note:

L(S) is a vector space.

Definition:

Linear independent:

A set consist of vectors x 1 ,x 2 ,.. .,x k is

said to be linearly independent if x 1 c 1

+ x 2 c 2 +... + x k c k = 0 c 1 = c 2 = ....

c k = 0 .Otherwise {x 1 ,x 2 ,...,x k } are

dependent.
Definition:

If a vector space X contains an

independent set of r vectors but

contains no independent set of r+1

vectors , then X has dimension r and

we write dim X = r.

Definition:

An independent subset of a vector

space X which spans the space X is

called a basis of X.

Note(i):

n
Let R be the set of all ordered n-

tuples x = (x 1 ,x 2 ,...,x n ) where

x 1 ,x 2 ,...,x n are real numbers and the


n
element of R called points or vectors

. Define x + y = (x 1 +y 1 ,x 2 +y 2 ,...
,x n +y n ) where y = (y 1 ,y 2 ,...,y n ) and

x = (x 1 ,x 2 ,..., x n ). Then x + y,
n
x R .

n
(ie) R is closed under addition and

scalar multiplication.

n
R is a vector space over the field

R.

Note(ii):

Let e1 = (1,0,0,...,0), e2 =

(0,1,0,...,0), .... e n = (0,0,0,...,1).

n
If x R , x = (x 1 ,x 2 ,...,x n ) , then x =

x j e j . We shall call {e 1 ,e 2 ,...,e n } the


n
standard basis of R .
Note(iii):

If a subset S = {v 1 ,v 2 ,...,v k } of X

contains the zero vector , then the

set S is linear dependent.

In particular if v1 = 0, then 1

v 1 +0v 2 +.. .+0v k = 0 1 0.

The set S is linearly dependent set.

Note(iv):

Let S = {v 1 ,v 2 ,...,v k } be linearly

dependent iff there exist a vector in

S which is a linear combination of

remaining vectors in S.
Note(v):
The set {v} consist of single vectors

is linearly independent iff the vector

v 0.

Note(vi):

If the set S is linearly independent

then any non-empty subset of S is

also linearly independent.

Definition:

If V is a vector space over a field F

and if W V then W is a subspace of

V if W itself is a vector space over F.

Theorem 4.1.1:

Let r be a positive integer. If a vector

space X is spanned by a set of r

vectors, then dim X r.


Proof:

Suppose dim X > r.

Let dim X = r + 1.

Therefore there is a vector space X

which contains an independent set

Q = {y 1 ,y 2 ,...,y r+1 } and which is

spanned by a set S 0 consisting r

vectors {x 1 ,x 2 ,...,x r }.

Suppose 0 i < r and suppose a set

S i has been constructed which spans

X and which consist of all y j with 1

j i plus a certain collection of r

i members of S 0 say {x 1 ,x 2 ,...,x ri }

(In other words, S i is obtained from

S 0 by replacing i of its elements by

members of Q, without altering the


span.) Since S i spans X, y i+1 is in

the span of S i and hence there are

scalars a 1 ,a 2 ,...,a i+1 ,b 1 ,b 2 ,...,b ri

with a i+1 = 1, such that

i+1 ri

a y +b x
j j k k = 0.
j=1 k=1

If all bks were 0, the independence

of Q would force all a j s to be zero,

which is a contradiction.

It follows that some x k S i is a linear

combination of the other members of

S i {y i+1 }.

Let T i = S i {y i+1 }.

Remove this x k from T i and call the

remaining set S i+1 .Then S i+1 spans


the same set as T i , namely X, (ie)

L(S i+1 ) = X, and S i has the properties

postulated for S i with i + 1 in place of

i.

Starting with S i , we have constructed

sets S 1 ,S 2 ,...,S r and Sr consist of

y 1 ,y 2 ,...,y r and L(S r ) = X. But Q is

linearly independent and hence y r+1

L(S r ), which is a contradiction to dim

X > r.

dim X r.

n
Cor: dim R = n.

Proof:

n
Since {e 1 ,e 2 ,.. .,e n } spans R , the
n
above theorem shows that dim R
n. Since {e 1 ,e 2 ,.. .,e n } is
n
independent, dim R n.

n
Therefore dim R = n.

Theorem 4.1.2:

Suppose X is a vector space and dim

X = n.

a. A set E of n vectors in X spans X

iff E is independent.

b. X has a basis, and every basis

consist of n vectors

c. If 1 r n and {y 1 ,y 2 ,...,y r }

is an independent set in X, then

X has a basis containing

{y 1 ,y 2 ,...,y r }.

Proof:

a. Suppose E = {x 1 ,x 2 ,...,x n }.
If E is independent then to prove

that L(E) = X.

Let yX.

Then A = {x 1 ,x 2 ,...,x n ,y} is

dependent. ( dim X = n )

a vector in A which is a

linear combination of remaining

vectors. Since E is independent,

no vector in E is a linear

combination of preceding

vectors.

y is a linear combination of

{x 1 ,x 2 ,...,x n }.

y L(E).

X L(E).

But L(E) X. Therefore X = L(E).

Conversely, let X = L(E).


Now to prove that E is

independent.

Suppose E is dependent. Then

one of the elements, say, {x k } is

a linear combination of preceding

vectors.

(i.e.) we can eliminate x k without

changing the span of E.

Hence E cannot span X, which is

a contradiction to L(E) = X.

Therefore E is independent.

b. Since dim X = n, X contains an

independent set of n vectors.

(i.e.) E = {x 1 ,x 2 ,...,x n } is

independent set in X.

By (a), L(E) = X.
X has a basis consists of n

vectors.

c. Let {x 1 ,x 2 ,...,x n } be a basis of X.

By hypothesis, {y 1 ,y 2 ,...,y r } is

an independent set in X.

the set

{y 1 ,y 2 ,...,y r ,x 1 ,x 2 ,...,x n } spans

X and is dependent, since it

contains more than n vectors.

one of the X i s is a linear

combination of the other

members of S.

If we remove this x i from S, the

remaining set still spans X. This

can be repeated r times and

leads to the basis of X which

contains {y 1 ,y 2 ,...,y r }.
Definition:

A mapping A of a vector space X

into a vector space Y is said to be

linear transformation if A(x 1 +x 2 )

= Ax 1 +Ax 2 ,

A(cx) = cA(x), for all x 1 ,x 2 ,xX and

all scalars c.

Definition:

Linear transformations of X into X is

called linear operators on X. If A is a

linear operator on X which

i. is one-to-one and

ii. maps X onto X, we say that A is

invertible.
1 1
We define A on X that, A (Ax) =
1
x or A(A x) = x.

Note:

i. If A is linear then A.0 = 0.

ii. If the set of n vectors say

{x 1 ,x 2 ,...,x n } is a basis of X then

any

element xX has a unique

representation of the function


n

x= cx i i and the linearity of A


i=1

allows us to compute Ax from the

vectors Ax 1 ,Ax 2 , ...,Ax n and the

coordinates c 1 ,c 2 ,... ,c n by the

formula
n

Ax = c Ax i i
i=1

Theorem 4.1.3:

A linear operator A on a finite-

dimensional vector space X is one-to-

one iff the range of A is all of X.

Proof:

Let {x 1 ,x 2 ,. . .,x n } be a basis of X.

Let (A) = {Ax/xX} =

{ }
n

c Ax / x X
i i i =L({Ax1, Ax2, ....Axn})
i=1

Now (A) = L(Q) where Q =

{Ax 1 ,Ax 2 ,...,Ax n }.

Now to prove that A is one-to-one iff


(A) = X.
(i.e.) to prove that A is one-to-one iff

L(Q) = X.

By theorem 4.1.2.(a), it is enough to

prove that A is one-to-one iff Q is

independent.

Suppose A is one-to-one .

( )
n n

Let c Ax = 0 i i A cx i i =0
i=1 i=1

( )
n n

A cx i i = A. 0 cx =0 i i
i=1 i=1

( )
c1 = c2 = .... cn = 0 since {x1, x2, ...., xn} is independent
Q is independent.

Conversely, let Q be independent and

( )
n

A cx i i =0
i=1

Then c Ax = 0
i i c1 = c2 = ....cn = 0 (Q is independent)
i=1

Therefore Ax = 0 only if x = 0.

Now, Ax = Ay A(x y) = 0 x y

= 0x = y.

Therefore A is one-to-one.

Definition:

Let L(X,Y) be the set of all linear

transformations of the vector space X


into the vector space Y. If Y = X ,

then we write L(X,Y) = L(X).

Definition:

If A 1 ,A 2 L(X,Y) and if c 1 ,c 2 are

scalars, define c 1 A 1 +c 2 A 2 by

(c 1 A 1 +c 2 A 2 )x = c 1 A 1 x+c 2 A 2 x (xX).

It is clear that c 1 A 1 +c 2 A 2 L(X,Y).

Definition:

If X,Y,Z are vector spaces and if

A,BL(X,Y) we denote their product

BA to be the composition of A and B :

(BA)x = B(Ax) (xX).

Then BA L(X,Z).
Note that BA need not be the same as

AB, even if X = Y = Z

Definition:

n m
For AL(R ,R ), define ||A|| = sup
n
{|Ax|/xR with |x|1} and the

inequality |Ax| ||A|| |x| holds for


n
all xR . Also if is such that |Ax|
n
|x| for all xR then ||A|| .

Theorem 4.1.4:
n m
a. If AL(R ,R ), then ||A|| <

and A is a uniformly continuous


n m
mapping of R into R .

n m
b. If A,BL(R ,R ) and c is a scalar

, then ||A+B|| ||A|| + ||B||,

||cA|| = |c| ||A||. With the


distance between A and B
n m
defined as ||A B||, L(R ,R ) is

a metric space.

n m n m
c. If AL(R ,R ) and BL(R ,R ),

then ||BA|| ||B|| ||A||.

Proof:

a. Let {e 1 ,e 2 ,...,e n } be the


n
standard basis in R and suppose

x = c i e i , |x| 1, so that |c i |

1 for i = 1,2,...,n. Then

|Ax| = |A( c i e i )|=| c i Ae i | |

Ci || Ae i | | Ae i | (since |c i |

1).

|Ax| | Ae i |< .

Therefore Sup|Ax| < .

(i.e.) ||A|| < .


Since |Ax Ay| = |A(x y)|
n
||A|| |x y| where x,yR .

Let > 0 be given.



Choose = > 0.
||A||

Now | x y | < | Ax Ay | ||A|| | x y | < ||A|| ||A|| =


Therefore A is uniformly

continuous.

n
b. ||A + B|| = sup{|(A+B)x|/xR

with |x|1}
n
= sup{|(Ax+Bx)|/xR with

|x|1}
n
sup {|Ax|+|Bx|/x R with

|x|1}
n
= sup{|Ax|/xR with |x|1} +
n
sup{|Ax|/xR with |x|1}

Double click this page to view clearly


= ||A|| + ||B||.

Hence ||A + B|| ||A|| + ||B||.

If c is a scalar, then ||cA|| =


n
sup{|(cA)x|/xR with |x|1}

n
= sup{|c|| Ax|/x R with

|x|1}
n
= |c| sup{|Ax|/xR with

|x|1}

= |c| ||A||
n m
Now to prove that L(R ,R ) is a

metric space.

Define d(A,B) = ||A B||, then

d(A,B) 0

d(A,B) = 0 ||A B|| = 0 A

= B.

Also d(A,B) = ||A B|| = ||B

A|| = d(B,A).
d(A,C) = ||A C|| where C
n m
L(R ,R )

= ||A B + B C||

|| A B|| + ||B C||

= d(A,B) + d(B,C)
n m
Therefore L(R ,R ) is a metric

space.

n m
c. Let A,B L(R ,R ).

Now ||BA|| = sup{|(BA)x|/x


n
R with |x|1}
n
= sup{|B(Ax)|/x R with

|x| 1}
n
sup{||B|| |Ax|/xR

with |x| 1}
n
= ||B|| sup{|Ax|/xR

with |x| 1}
= ||B|| ||A||.

Theorem 4.1.5:

Let be the set of all invertible linear


n
operator on R .

n
a. If A, BL(R ), and ||B
1
A||.||A || < 1, then B.

n
b. is an open subset of L(R ),
1
and the mapping A A is

continuous on .

Proof:
1
a. Put ||A || = 1/ and ||B A||

= .
1
Then ||B A||.||A || < l /

< 1 < .

Therefore > 0,
1
Now |x| = | A A(x)| = |
1 1
A (Ax)| ||A || |A(x)|
1
= |Ax| ( ||A || = 1/)

= |Ax Bx + Bx| |Ax

Bx| + |Bx| = |(AB)x| + |Bx|

||AB|| |x| + |Bx| = ||B

A|||x| + |Bx|

= |x| + |Bx|( ||B A||

= .)

|x| |x| |Bx| (

)|x| |Bx|

|Bx| ( )|x| > 0 (

> 0)

Bx 0.

Now Bx By Bx By 0

B(xy) 0 x y 0 x y.

B is one-to-one.
By theorem 4.1.3, B. This

holds for all B with ||B A|| <


n
and . is open subset of L(R ).

1
b. Replace x by B y in ( )|x|

|Bx|, we get
1 1
( )| B y | |BB y |
n
= |y| (yR )

1 1
| B y | |y|( ) .

1 1 n
|| B || =sup{| B y | / yR
1
} ( ) .
1 1 1 1 1
Now B A = B A A B
1 1 1
B A = B (A B)A .
1 1 1 1
||B A || = ||B (A B)A
1 1
|| || B || ||A B|| ||A ||
1
..

As B A, 0.
1 1
||B A || as 0.

1
A A is continuous on .

Definition: Matrices

Suppose {x 1 ,x 2 ,...,x n } and

{y 1 ,y 2 ,...,y m } are bases of vector

spaces X and Y respectively. Then

every AL(X,Y) determines a set of

numbers a ij such that


m

Axj = a y ij i (1 j n)
i=1

It is convenient to visualize these

numbers in a rectangular array of m

rows and n columns, called an m by

n matrix:
[ ]
a11 a12 a1n

a21 a22 a2n


[A] =

am1 am2 amn

Observe that the coordinates a ij of

the vector Ax j appear in the jth

column of [A]. The vectors Ax j are

called the column vectors of [A]. The

range of A is spanned by the column

vectors of [A].

( )
n n n n m

If x = c x then Ax = A c x
j j j j = c Ax = c a y
j j j ij j
j=1 j=1 j=1 j=1 i=1

[ ]
m n

= a c ij j yi.
i=1 j=1

{ a }
1/2
Next to prove that ||A|| ij2

Double click this page to view clearly


Suppose {x 1 ,x 2 ,...,x n } and

{y 1 ,y 2 ,...,y m } are the standard bases


n m
of R and R respectively. By

definition Ax = a c
i [ ] j
ij j yi

= [a i1c1 + ai2c2 + .... + a1ncn ] yi


i

= [a11c1 + a12c2 + .... + a1ncn]y1 + [a21c1 + a22c2+ .... + a2ncn] + y1....

+ [am1c1 + am2c2 + ... + amncn] yn.

( ) ( ) ( )
n n n

= a1jcj y1 + a2jcj y2 + .... + anjcj yn.


j=1 j=1 j=1

[ ] [ ]
2
m n m n n

|Ax| = a c
2
aijcj ij2 j2 (by Schwarz inequality)
i=1 j=1 i=1 j=1 j=1


2
= aij2 |x|
i, j

{ a }
1/2
Thus ||A|| ij2

Double click this page to view clearly


CYP QUESTIONS:

1. If S is a non-empty subset of a

vector space X, prove that the

span of S is a vector space.

2. Prove that BA is linear if A and B

are linear transformations. Prove


1
also that A is linear and

invertible.

3. Assume AL(X,Y) and Ax = 0

only when x = 0. Prvoe that A is

1 1.

SECTION-4.2
DIFFERENTIATION

If f is a real valued function with


1
domain (a,b) R and if x(a,b),

then f'(x) is usually defined to be the

real number
f(x + h) f(x)
lim h provided that this
h0

limit exists.

f(x + h) f(x)
(i. e.) f'(x) = lim h
h0

f(x+h) f(x) = f(x)h + r(h), where

the remainder r(h) is

small in the sense that


r(h)
lim h = 0.
h0

Definition:

1 m
If f is a map from (a,b) R to R is

differentiable mapping and x (a,b)

then f'(x) is the linear transformation

of
1 m
R to R that satisfies
f(x + h) f(x) f'(x)h
lim h =0
h0
Definition:

n
Suppose E is an open set in R and
m
f maps E into R and xE if there
n
exist a linear transformation A of R
m
into R such that
|f(x + h) f(x) Ah|
lim |h|
= 0, .....(1)
h 0

then f is differentiable at x and we

write f'(x) = A. If f is differentiable

at every xE, we say that f is

differentiable in E.

Theorem 4.2.1:

Suppose E and f are as in the above

definition , xE, and (1) holds with A

= A 1 and with A = A 2 . Then A 1 = A 2 .

Proof:

Let B = A 1 A 2 .
Now Bh = (A 1 A 2 )h = A 1 h A 2 h

= f(x+h) f(x) f(x+h) +

f(x) + A 1 h A 2 h

= [f(x+h) f(x) A 2 h]

[f(x+h) f(x) A 2 h]

|Bh| |f(x+h) f(x) A 1 h| +

|f(x+h) f(x) A 2 h|

|B(h)|
|h|

|f(x + h) f(x) A1h|
|h|
+
|f(x + h) f(x) A2h|
|h| 0

as h 0

|B(h)|
|h| 0 as h 0

For fixed h 0, it follows that

|B(th)|
|th| 0 as t 0 .....(2)
The linearity of B shows that the left

side of (2) is independent of t.

n
Thus Bh = 0 for every h R . Hence

B = 0.

Therefore A 1 = A 2 .

(Chain rule)

Theorem 4.2.2:

n
Suppose E is an open set in R , f
m
maps E into R , f is differentiable at

x 0 E, g maps an open set containing


k
f(E) into R , and g is differentiable

at f(x 0 ). Then the mapping F of E


k
into R defined by F(x) = g(f(x)) is

differentiable at x 0 , and F'(x 0 ) =

g'(f(x 0 ))f(x 0 ).
Proof:

Put y 0 = f(x 0 ), A = f(x 0 ), B = g'(y 0 ),

and define .

u(h) = f(x 0 +h) f(x 0 ) Ah,

v(k) = g(y 0 +k) g(y 0 ) Bk, for all


n m
h R and k R for which f(x 0 +h)

and g(y 0 +k) are defined.

Since f is differentiable at x 0 , we
|f(x0 + h) f(x0) Ah|
have lim |h|
=0
h0

|u(h)| |u(h)|
(i. e.) lim |h|
= 0. (i. e.) lim (h) = 0 where (h) = |h|
h0 h0

(h) 0 as h 0.

|||ly g is differentiable at x 0 , we have


|g(y0 + k) g(y0) Bk|
lim |k|
=0
k0

Double click this page to view clearly


|v(k)| |v(k)|
(i. e.) lim |k|
= 0. (i. e.) lim (k) = 0 where (k) = |k|
h0 h0

(k) 0 as k 0.

Given h, put k = f(x 0 +h) f(x 0 ).

Then |k| = |f(x 0 +h) f(x 0 )|

= |Ah + u(h)| |Ah| + |u(h)|

||A|| |h| + (h)|h|

(i.e.) |k| [||A|| + (h)] |h|.

Now F(x 0 +h) F(x 0 ) BAh

= g(y 0 +k) g(y 0 ) Bah

= Bk + v(k) BAh = B(k Ah) +

v(k) =

B(u(h)) + v(k).

|F(x 0 +h) F(x 0 ) Bah| = |B(u(h))

+ v(k)| |B(u(h))| + |v(k)|

Double click this page to view clearly


|F(x0 + h) F(x0) BAh|
|h|
||B|| (h)|h| + (k)| [||A|| + (h)] |h|.

Since (h) 0 as h 0 and (k) 0

as k 0, the RHS tends to

Zero
|F(x0 + h) F(x0) BAh|
(i. e.) lim |h|
=0
h0

F is differentiable at x 0 and F'(x 0 )

= BA.

(i.e.) F'(x 0 ) = g'(y 0 )f'(x 0 )

F'(x 0 ) = g'(f(x 0 )).f(x 0 ).

Definition:

Consider a function f that maps an


n m
open set E R into R Let

{e 1 ,e 2 ,...,e n } and {u 1 ,u 2 ,...,u m } be

Double click this page to view clearly


n m
the standard bases of R and R

respectively. The components of f are

real functions f 1 ,f 2 ,...,f m defined by


m

f(x) = f (x)ui i (x E) or,


i=1

equivalently, by f i (x) = f(x).u i , 1i

m.

For x E, 1 i m, 1 j n, we

define

fi(x + tej) fi(x)


(Djfi(x)) = lim t provided
t0

the limit exists, where D j f i is the

derivative of f i with respect to X j ,

keeping the other variables fixed, we


fi
denote xj in place of D j f i , and D j f i is

called the partial derivative.


Theorem 4.2.3:

Suppose f maps an open set E


n m
R into R ,and f is differentiable

at a point x E. Then the partial

derivatives (D j f i )(x)

exist, and
m

f'(x)ej = (D f )(x)u j i i (1 j n)
i=1

Proof:

Fix j. Since f is differentiable at x,

f(x + te j ) f(x) = f'(x)(te j ) + r(te j )

where |r(te j )|/t 0 as t 0.

The linearity of f(x) shows that


f(x + tej) f(x)
lim t = f'(x)ej. .....(1)
t0
If we represent f interms of its

components, then (1) becomes

m
fi(x + tej) fi(x)
t0
lim t
ui = f'(x)ej.
i=1

m
fi(x + tej) fi(x)
(i. e.) lim t
ui = f'(x)ej.
t 0
i=1

(D f )(x)u = f'(x)e
j i i j
i=1

Therefore the partial derivatives of

(D j f i )(x) exist, and

f'(x)ej = (D f )(x)u (1 j n).


j i i
i=1

Note : By the above theorem,


m

f'(x)ej = (D f )(x)u j i i (1 j n).


i=1
m m

f'(x)e1 = (D f )(x)u , f'(x)e = (D f )(x)u , ....,


1 i i 2 2 i i
i=1 i=1

f'(x)ej = (D f )(x)u .
j i i
i=1

Let [f(x)] be the matrix that

represent f(x) with respect to the

standard bases. Then

[ ]
(D1f1)(x) (Dnf1)(x)
(D1f2)(x) (Dnf2)(x)
[f'(x)] =

(D1fm)(x) (Dnfm)(x)

Theorem 4.2.4:

Suppose f maps a convex open set


n m
E R into R , f is differentiable in
E, and there is a real number M such

that ||f'(x)|| M for every x E.

Then |f(b) f(a)| M|b a| for all a

E, bE.

Proof:

Fix aE and bE.

Define (t) = (1 t)a + tb, for every


1
t R such that (t) E.

Since E is convex, 0 t 1 implies

(t)E.

Put g(t) = f((t)).

g'(t) = f'((t)). '(t) = f'((t)) (b

a).

|g'(t)| = | f((t)) (b a) | ||

f((t))|| |b a| M |b a|.
When t = 0, g(0) = f((0)) = f(a).

When t = 1, g(1) = f((1)) = f(b).

By mean value theorem for vector-

valued functions, we have

|g(1) g(0)|(10)g'(t)

(i.e.) | f(b) f(a) | M (b a)

Cor: If, in addition, f(x) = 0 then f is

constant.

Proof:

To prove this, note that the

hypothesis of the theorem holds with

M = 0.
A differentiable mapping f of an open
n m
set E R into R is said to be

continuously differentiable in E, if fis

a continuous mapping of E into


n m
L(R ,R ).

(i.e.) for every xE and to every >

0, corresponds a > 0 such that

|| f(y) f(x)|| < if yE and |x

y| < .

If this so, we also say that f is a ? -

mapping , or that f ? (E).

Theorem 4.2.5:

n
Suppose f maps an open set ER
m
into R . Then f ? (E) iff the partial

derivatives Djfi exist and are


continuous on E for 1 i m, 1 j

n.

Proof:

Assume frist that f ? (E).

By theorem 4.2.3, we have f(x)e j .u i

= (D j f i )(x)for all i and j and for all

xE.

Now (D j f i (y) (D j f i (x) = f(y)e j .u i

f(x)e j .u i

= (f'(y)

f(x))e j .u i .

Since {e 1 ,e 2 .....e n } and

{u 1 ,u 2 ,...,u m } are the standard


n
bases of R and
m
R respectively, | u i | = | e j | = 1.

| (D j f i (y) (D j f i )(x)| = |(f'(y)

f'(x))e j .u i |

= |(f'(y)

f(x))e j | ||f(y) f(x)|| |e j |

= ||f(y)

f(x)||

D j f i is continuous.

Conversely let D j f i be continuous.

Consider the case m = 1. (i.e.) f = f 1 .

Fix xE and > 0.

Since E is an open set, there is an

open ball SE with center at x and

radius r.
Since D j f i is continuous, r can be so

chosen that

|(Djfi)(y) (Djfi)(x)| < n .....(1) (y S, )


1 j n

Suppose
n

h= h e with |h| < r, put v


j j 0 = 0, and
j=1

v k = h 1 e 1 +h 2 e 2 + .... +

h k e k for 1 k n.

Then f(x+h) f(x) = f(x + v n ) f(x

+ v0)

= f(x + v 1 ) f(x + v 0 ) + f(x + v 2 )

f(x + v 1 ) + f(x + v 3 ) f(x + v 2 )

+......+ f(x + v n ) f(x + v n1 )


n

= [f(x + v ) f(x + v
j j1 )] .....(2)
j=1

Since | v k | < r for 1 k n and

since S is convex, the segments with

end points f(x + v j1 ) and f(x + v j )

lie in S.

Since v j = h 1 e 1 +h 2 e 2 + .... + h j e j and

v j1 = h 1 e 1 +h 2 e 2 + .... + h j1 e j1 we

have v j v j1 = h j e j v j = v j1 +

hjej,

Apply Mean value theorem to the jth

summand in (2),

f(x + v j ) - f(x + v j1 ) = [(x + v j ) (x

+ v j1 )] (f(x + v j1 + j h j e j ))
where 0 < j < 1 and this differs from

h j D j f(x) by less than |h| n , using (1)

f(x + v j ) f(x + v j-1 ) = [v j v j1 ]

D j f(x + v j1 + j h j e j )

= h j D j f(x +

v j1 + j h j e j )

Substitute in (2), we get

f(x + h) f(x) = h D [f(x + v


j j j1 + jhjej) ]
j=1

f(x + h) f(x) h (D f)(x) j j


j=1

n n

= h D [f(x + v
j j j1 + jhjej) ] h (D f)(x)
j j
j=1 j=1

| |
n

f(x + h) f(x) h (D f)(x) j j


j=1

Double click this page to view clearly


| |
n n

= h D [f(x + v
j j j1 + jhjej) ] h (D f)(x)
j j
j=1 j=1

| |
n

= [h [D [f(x + v
j j j1 ]
+ jhjej) Dj[f(x)] ]]
j=1

|h | |D [f(x + v
j j j1 ]
+ jhjej) Dj[f(x)] |
j=1

n

< |hj| n (by (1))
j=1

|h|

|
n

f(x + h) f(x) h (D f)(x)


j j
j=1
|h|
<

f is differentiable at x.

f'(x)ej = (D f )(x)u j i i , which is


i=1

nothing but jth column vector

Double click this page to view clearly


[ ]
(D1f1)(x) (Dnf1)(x)
(D1f2)(x) (Dnf2)(x)
on [f'(x)], where [f'(x)] =

(D1fm)(x) (Dnfm)(x)

n n n

If h = h e then f'(x)h = f'(x) h e = f '(x)h e


j j j j j j
j=1 j=1 j=1

n m

= h (D f )(x)u
j j i i
j=1 i=1

Since m = 1, f'(x)h = h (D f)(x)


j j
j=1

The matrix [f(x)] consists of the row

(D 1 f)(x).....(D n f)(x); and since

D 1 f,....., (D n f) are continuous

functions on E,f ? (E).

CYP QUESTIONS:

1. If f(0,0) = 0 and
xy
f(x, y) = 2 2 if f(x, y) (0, 0)
x +y

Double click this page to view clearly


Prove that (D 1 f)(x,y) and

(D 2 f)(x,y) exist at every point of


2
R , although f is not continuous

at (0,0).

2. Suppose that f is a real-valued

function defined in an open set


n
E R , and that the partial

derivatives are bounded in E.

Prove that f is continuous in E.

SECTION-4.3 THE
CONTRACTION PRINCIPLE.
Definiton:
Let X be a metric space, with metric

d. If maps X into X and if there is a

real number c < 1 such that d((x),

(y)) cd(x,y) for all x,y X, then

is said to be a contraction of X into

X.
Theorem 4.3.1: (Contraction
mapping theorem)

If X is a complete metric space, and

if is a contraction of X into X, then

there exists one and only one x X

such that (x) = x.


Proof:

Let x 0 X. Define {x n }by setting

(x n ) = x n+1 .

First to prove that {x n } is a Cauchy

sequence in X.

Since is a contraction, there exists


a number c such that 0 < c <1 such

that d((x), (y)) < cd(x,y) for all

x,y X

For n 1, we have d(x n+1 , x n ) =

d((x n ), ( x n1 )) c d(x n , x n1 )

cd((x n1 ), ( x n2 )) =
c.c
2
d(x n1 ,x n2 ) = c d(x n1 ,x n2 ) =....
n
d(x n+1 , x n ) c d(x 1 ,x 0 ) .

if n < m, then it follows that

d(x n , x m ) d(x n , x n+1 ) + d(x n+l ,

x n+2 ) +.....+ d(x m1 , x m )

n n+
c d(x 1 ,x 0 ) + c 1d(x 1 ,x 0 )
m1
+.....+ c d(x 1 ,x 0 ).

n 2 mn1
c d[1 + c + c +....+ c ]

d(x 1 ,x 0 )

n 2
< c d[1 + c + c +....] d(x 1 ,x 0 )

n
d(x1, x0)
c
< 1c

n
Since c < 1, (c ) 0 as n
Therefore, given > 0, there exists a

positive integer n such that

n
d(x1, x0) <
c
1c

d(x n , X m ) <

{x n } is a Cauchy sequence in X.

Since X is complete metric space,

{x n } x.

Since is contraction, is

continuous.

(x n ) (x).

(i. e.) (x) = lim (xn) = lim xn + 1 = x


n n

x is a fixed point of X.

Now to prove the uniqueness:


Suppose y X such that y x and

(y) = y.

Now d(x,y) = d((x), (y)) cd(x,y)

(i. e.) ( 1 c )d(x, y) 0 .....(1)

Since c < 1, we have 1 c > 0 and

since d(x,y) 0,

(1 c)d(x,y) 0, which is a

contradiction to (1).

x is a unique fixed point of X,


SECTION-4.4 THE INVERSE
FUNCTION THEOREM

THE INVERSE FUNCTION THEOREM

Theorem 4.4.1:

Suppose f is a ? mapping of an open


n
set E R into

n
R , f(a) is invertible for some aE

and b = f(a) then

a. there exist open sets U and V in


n
R such that aU, bV, f is one-

to-one on U, and f(U) = V;

b. if g is the inverse of f [which

exists, by (a)], defined in V by

g(f(x)) = x, xU, then g C (V)

Proof:

a. Put f(a) = A.
Given that f(a) is invertible.
1
A is invertible (i.e.) A exists.
1
Choose so that = .
2 || A | |
1

Since f is continuous at a , there

exists an open ball UE with

center at a such that ||f(x)

f(a)|| <

(i. e.) ||f'(x) A|| < . .....(*)


Define a function by, (x) = x
1
+ A (y f(x)) where xE and
n
y R .

Note that f(x) = y iff x is a fixed

point of .

Let f(x) = y. Then (x) = x +


1 1
A (y y) = x + A (0) = x.

x is a fixed point of .
Conversely, let (x) = x.
1
Then x = x + A (y f(x))
1
A (y f(x)) = 0 yf(x) = 0

y = f(x).
1
Now '(x) = I + A (0 f'(x)) =
1 1 1
A A A f(x) = A (A f(x))

||'(x)|| = ||A (A f'(x))|| ||A || |A f'(x)| <


1 1 1 1
2 . = 2 . .....(1)

By Mean value theorem for single

variable,

| (x 1 ) (x 2 )| = |x 1 x 2 |

||'(x)||, x(x 2 , x 1 )

|(x1) (x2)| < 12 |x1 x2| where x1, x2 U .....(2)

is a contraction.

By using fixed point theorem(

theorem 4.3.1) , the function

has atmost one fixed point in U

Double click this page to view clearly


so that f(x) = y for at most one

xU.

By theorem 4.1.3, f is one-to-

one in U.

Put V = f(U).

Let y 0 V. Then y 0 = f(x 0 ) for

some x 0 U.

Let B be an open ball with center

at x 0 and radius r > 0, so small



that its closure B lies in U.

Now to prove that V is open.

(i.e.) to prove that |y y 0 | < r

yV.

1
Since (x 0 ) = x 0 + A (y

f(x 0 )),
1
(x 0 ) x 0 = A (y f(x 0 ))
|(x0) x0| = |A 1 (y f(x0))| ||A 1 || |y f(x0)| 1
2 |y y0|

1 r
< 2 r = 2 ......(3)

If x B , then |(x) x 0 | = |(x)

(x 0 ) + (x 0 ) x 0 |

|(x)

(x 0 )| + |(x 0 ) - x 0 |
1 r
<
2 | x x0| +
2
r r
< + =r
2 2
|(x) x 0 | < r.

(x)B(x 0 ,r).

|
Also if x1, x2 B, then (x1) (x2) < | 1
2 |x1 x2| (by (2))


is a contraction of B into B .

n
Since R is complete and B is

closed, B is complete.( since any

closed subset of a complete

metric space is complete).

Double click this page to view clearly


By the above theorem, has a

fixed point, x B

y = f(x) f(B) f(U) = V

yV.

(i.e.) y is an interior point of V.

y is open set.

b. Now to prove that g? (V).

Fix yV and y + k V. Then there

exist xU, x + hU so that

y = f(x); y + k = f(x + h)

Now, (x + h) (x) = x + h +
1 1
A (y f(x + h)) x A (y

f(x))
1
= h + A (y f(x

+ h) y + f(x))

(f(x + h) f(x)) = h
1
=h + A
A 1
k .....(4)
Now |(x + h) (x)|
1
2 |h| |h A 1
|
k
1
2 |h|

1
2 |h| |h A k| |h| |A k|
1 1

|A 1
k | 1
2 |h| 2 |A 1
|
k |h|

|h| 2 |A k| 2 |A | |k|
1 1
| | 1
|k|. .....(5)

By (*), ||f'(x) A|| < . ||f'(x) A|| < || 2 A


1
1
||

|| A
1
|| ||f'(x) A|| < 21 < 1.

By theorem 4.1.5, f(x) is invertible

linear operator.

(i.e.) f(x) has an inverse, say T.

1
(i. e.) T = f'(x) .

Now g(y+k) g(y) Tk = g(f(x+h))

g(f(x)) T

= x+h x

Double click this page to view clearly


Tk = h Tk

= T[k h/

T] = T[f(x+h) f(x) hf(x)]

|g(y + k) |
g(y) T(k)| = T[f(x + h) f(x) hf'(x)] |

||T|| |f(x + h) f(x) hf'(x)|

|g(y + k) g(y) Tk| |f(x + h) f(x) hf'(x)|


|k|
||T || |k|

|f(x + h) f(x) hf'(x)|


||T || (h) (by (5))

Since f is differentiable, the RHS of

the inequality tends to zero.

Clearly, g is differentiable and g'(y)

= t.

But T was chosen to be the inverse of

f (x) = f(g(y)),

Double click this page to view clearly


[ ]
1 1
g'(y) = [f'(x)] = f'(g(y)) . .....(6)

Since g is a continuous mapping of V

onto U and f is a continuous mapping

of U into the set of all invertible


n
elements of L(R ), and that inversion

is a continuous mapping of onto

(by theorem 4.1.3) By (6) g ? (V)

Theorem 4.4.2: If f is a ? mapping


n n
of an set B R into R and if f(x) is

invertible for every xE, then f(W) is

n
an open subset of R for every open

set WE.

Proof:

Let y f(W). Then there exist an

element xW such that f(x)= y Since

W E, xE.
By hypothesis, f(x) is invertible.

By the inversion function theorem,

there exists an open set U and V in

n
R such that xU and yV, f is one-

to-one and f(U) = V.

Since W is open and x W, U can be

selected so that U W.

Therefore f(U) f(W).

(i.e,) V f(W).

But f(x) V

(i.e,) yV and V is open, there exists

a neighborhood Nr(y)V,

But V f(W). Ttherefore Nr(y)

f(W).

n
Therefore f(W) is an open in R ,
CYP QUESTIONS:

1. Suppose that f is a differentiable

real function in an open set E


n
R , and that f has a local

maximum at a point xE. Prove

that f(x) = 0

2. If f is a differentiable mapping of
n
a connected open set E R into
m
R , and if f'(x) = 0. for every

xE, prove that f is constant in

E.
UNIT-5

Unit Structure

Section 5.1: The implicit function

theorem

Section 5.2: The rank theorem

Section 5.3: Determinants.

Section 5.4: Derivatives of

higher order

Section 5.5: Differentiation of

integrals

Introduction

In this unit we shall discuss about the

implicit function theorem, the rank

theorem, determinants, derivatives


of higher order and differentiation of

integrals.

SECTION-5.1 THE IMPLICIT


FUNCTION
THEOREM

If f is a continuously differentiable

real function in the plane, then the

equation f(x,y) = 0 can be solved for

y interms of x in a neighborhood of

any point (a,b) for which f(a,b) =


f
0 and y 0 . The preceding very

informal statement is the simplest

case of the so-called implicit

function theorem.

n
Notation: If x = (x 1 ,x 2 ,...,x n )R
m
and y = (y 1 ,y 2 ,...,y m )R , let us

write (x,y) for the point (or vector)


(x 1 ,x 2 ,...,x n ,
n+m
y 1 ,y 2 ,...,y m )R .

Every linear transformation


n+m n
AL(R , R ) can be split into two

linear transformations A x and A y ,

defined by A x h = A(h,0), A y k =
n m
A(0,k) for any hR , kR . Now to
n n+m n
show that A x L(R ), A y L(R , R )

and A(h,k) = A x h + A y k.

proof:
n
i. If h 1 ,h 2 R then Ax(h 1 + h 2 ) =

A(h 1 + h 2 ,0) = A[(h 1 ,0), (h 2 ,0)]

= A[(h 1 ,0)] +

A[(h 2 ,0)]

=A x h 1 + A y k 2 .
ii. A x (ch) = A(ch,0) = A[c(h,0)] =

cA(h,0) = c A x h where c is a

scalar.

A x is a linear.

|||ly A y is a linear.

iii. A x h + A y k = A(h,0) + A(0,k) =

A[(h,0) + (0,k)] = A(h,k)

n+m n
Theorem 5.5.1: If A L(R , R )

and if A x is invertible, then there


m
corresponds to every kR a unique
n
hR such that A(h,k) = 0. This h can

be computed form k by the formula h


1
= (A x ) A y k.

Proof:

Given that A(h,k) = 0.


Since A(h,k) =A x h + A y k, A x h + A y k

= 0. A x h = A y k.

-1
Since A x is invertible, A x exists.

1 1 1
(i.e.) A x A x = Ax AX= I Ax Ax
1
h = (A x ) Ayk

1
h = (A x ) A y k.

Now to prove the uniquness.

n
Suppose h 1 ,h 2 R such that h 1 =
1 1
(A x ) A y k 1 ., h 2 = (A x ) Ayk2

then h 1 h 2 = 0. Therefore h is

unique.
(Implicit Function Theorem)

Theorem 5.5.2:

Let f be a ? -mapping of an open set


n+m n
E R into R ,such that f(a,b)

= 0 for some point (a,b)E. Put A

= f(a,b) and assume that A x is

invertible. Then there exist open sets


n+m m
U R and W R , with (a,b)U

and bW, having the property:

To every y W corresponds a unique

x such that (x,y) U and f(x,y) = 0.

If this x is defined to be g(y), then g


n
is a ? mapping of W into R , g(b) =

a, f(g(Y),y) = 0 (yW), and g'(b) =


1
(A x ) Ay.

[This g is called implicit function].


Proof:

n+m
Define F: E R by F(x,y) =

(f(x,y),y) for every (x,y) E

Then F is a ? -mapping of E into


n+m
R .

Now to prove that F is differentiable

at (x,y) E. and F is continuous map


n+m
of E into R .

n m
Let (x,y) E and let h R , k R

such that (x+h,y+k)E,

F(x+h,y+k) F(x,y) =

(f(x+h,y+k),y+k) (f(x,y),y)

= (f(x+h,y+k) f(x,y), k)

= (f'(x,y)(h,k) + r(h,k), k+0),


where lim
|r(h, k)| = 0
(h, k) 0
(h, k)

F(x+h,y+k) F(x,y) = (f(x,y)(h,k)

,k) + (r(h,k), 0).

Therefore F is differentiable at (x,y)

E.

Let > 0 be given.

Since fis continuous on E, there

exists a > 0 such that

|| f(x,y) f(u,v) || < whenever

|(x,y) (u,v)| < .

Now F'(x,y)(h,k) F'(u,v)(h,k) =

(f(x,y)(h,k) ,k) (f(u,v)(h,k),k)


= ((f(x,y)

f(u,v))(h,k),0)

|| F'(x,y)(h,k) F'(u,v)(h,k) || ||

f(x,y) f(u,v)|| |h,k| < if |h,k|1.

| F'(x,y)(h,k) F'(u,v)(h,k) || < .

F is a ? -mapping of an open set E


n+m
into R .

Next to prove that F'(a,b) is an


n+m
invertible element of L(R ).

Since f(a,b) = 0, we have f(a+h,b+k)

f(a,b) = f(a,b)(h,k)+r(h,K),

|r(h, k)|
where lim =0
|(h, k)|
(h, k) 0
f(a+h,b+k) = A(h,k) + r(h,k)

(since f(a,b) = 0)

Since F(a+h,b+k) F(a,b) =

(f(a+h,b+k),b+k) (f(a,b),b)

(f(a+h,b+k),k)

(A(h,k)+r(h,k),k)

(A(h,k),k) + (r(h,k),0)

n+m
Fis the linear operator on R

that maps (h,k) to (A(h,k),k).

(i-e. ) F'(a,b)(h,k) = (A(h,k),k).


Suppose F'(a,b)(h,k) = 0 then

(A(h,k),k) = 0 A(h,k) = 0 and k =0

(i.e.) A(h,0) = 0 A x h = 0.

1
Since A x is invertible, A x exists .

1 1
Ax (A x h) = 0 (A x A x )h = Ih

= 0 h = 0.

F'(a,b)(h,k) = 0 (h,k) = (0,0).

n+m
F'(a,b) is a one-to-one on L(R ).

n+m
Since R is a finite dimensional

vector space and by theorem 4.1.3,

F'(a,b) is onto, we have F'(a,b) is

invertible.
Now apply the inverse function

theorem to F.

It shows that there exist open sets U


n+m
and V in R , with (a,b)U and

F(a,b) V such that F is a 1 1

mapping of U onto V.

Now F(a,b) = (f(a,b),b) = (0,b)V .

m
Let W = {y R /(0,y) V}.

Since (0,b) V, bW.

If yW then (0,y) V = F(U).

(0,y) = F(x,y), x,y U

(0,y) = (f(x,y),y)

f(x,y) = 0.
Now to prove the uniqueness.

Suppose, with the same y, that

(x,y)U and f(x,y) = 0.

F(x,y) = (f(x,y),y) = (0,y) =

(f(x,y),y) = F(x,y)

Since F is 1 1,F(x,y) = F(x,y)

(x,y) = (x,y) x= x.

Next to prove the second part.

Define g(y) = x, for y W such that

(g(y),y)U and f(g(y),y) = 0.

( )
Consider F(g(y), y) = f(g(y), y), y = (0, y) .....(1)

Since F is 1 1 on U and F(U) = V, G


1
= F exists.
By inverse function theorem, G ?

(V).

(1) gives G(F(g(y),y)) = G(0,y)

(g(y),y) = G(0,y) ? (V)

g ? (W) (or) g? .

1
Next to prove g'(b) = (Ax) Ay.

Put (y) = (g(y),y).

m
Let k R such that y+k W.

Now (y+k) (y) = (g(y+k),y+k)


(g(y),y)

= (g(y+k)

g(y),k)
= (g'(y)k +

r(k),k).

= (g'(y)k,k) +

(r(k),0)

is differentiable and '(y)k = (g'(y)


k, k) .....(2)

Since (y) = (g(y),y), f((y)) =

f(g(y),y) = 0 in w.

The chain rule therefore shows that

f((y)) '(y) = 0

When y = b,f'((b)) '(b) = 0.

Therefore (y) = (g(y),y) = (a,b)

and f'((y)) = A.

Thus f'(a, b)'(b) = 0. (i. e.) A'(b) = 0.


.....(3)
Consider A x g'(b)k+ A y k = A(g'(b)k,k)

= A'(b)k (by (2))

A x g'(b)k+A y k = 0 (by (3))

A x g'(b) +A y = 0 Axg'(b) = - A y .

-1
Since A x is invertible, A x exists.

-I
A x A x g'(b) = - (A x -1)Ay.

1
g'(b) = (Ax )A y .

Example:

Take n =2 , m =3 and consider the


5 2
mapping f = (f 1 ,f 2 ) of R into R

given by

x1
f 1 (x 1 ,x 2 ,y 1 ,y 2 ,y 3 ) = 2e + x 2 y 1 4y 2

+ 3
f 2 (x 1 ,x 2 ,y 1 ,y 2 ,y 3 ) = x 2 cos x 1 6x 1 +

2y 1 y 3 .

If a = (0,1) and b = (3,2,7), then

f(a,b) = 0.

The matrix of the transformation A =

f'(a,b) is given by

[ ]
D1f1 D2f1 D3f1 D4f1 D5f1
[A] =
D1f2 D2f2 D3f2 D4f2 D5f2

[ ]
2 3 1 4 0
=
6 1 2 0 1

[ ] [ ]
2 3 1 4 0
Hence [Ax] = and [Ay] = .
6 1 2 0 1

We see that the column vectors of

[A x ] are independent, A x is invertible

and the implicit function theorem


asserts the existence of a mapping

g , defined in a neighborhood of

(3,2,7), such that g(3,2,7) = (0,1)

and f(g(y),y) = 0.

[ ]
1 3
[( A ) ] = [ A ]
1 1 1 1
Since x x = Adj A =
|Ax| 20
6 2

[ ][ ]
1 1 1 3 1 4 0
g'(3, 2, 7) = Ax Ay = 20
6 2 2 0 1

[ ]
1 1 6 4 3
= 20
6 + 4 24 2

[ ][ ]
1 5 4 3 1/4 1/5 3 / 20
= 20 =
10 24 2 1/2 6/5 1 / 10

Interms of partial derivatives, the

conclusion is that
1 1 3
D1g1 = 4 D2g1 = 5 D3g1 = 20

1 6 1
D1g2 = 2 D2g2 = 5 D3g2 = 10 at the point (3, 2, 7)

Double click this page to view clearly


CYP QUESTIONS:

1. Take n = m =1 in the implicit

function theorem , and interpret

the theorem graphically.

SECTION-5.2 THE RANK


THEOREM

Definition:

Suppose X and Y are vector spaces,

and AL(X,Y). The null space of A, ?

(A) , is the set of all xX at which Ax

= 0.

(i.e.) ? (A) = { xX/Ax = 0}.

? (A) is a vector space in X.

The range of A, (A) = {Ax/xX}.

Result 1: ? (A) is a vector space in

X.
Let x 1 ,x 2 ? (A). Then Ax 1 = 0, Ax 2

= 0.

Now A(x 1 +x 2 ) = Ax 1 + Ax 2 = 0 + 0 =

0.

X 1 + X 2 ? (A).

Also A(cx) = cAx = c0 = 0, where c is

a scalar.

cx ? (A).

? (A) is a vector space in X.

Result 2 : (A) is a vector space in

X.

Let y 1 ,y 2 (A). Then there exist

x 1 ,x 2 X such that
y 1 = Ax 1, y 2 = Ax 2 .

Now A(x 1 +x 2 ) = Ax 1 + Ax 2 = y 1 +y 2 .

x 1 + x 2 (A).

If y . (A), then there exists xX

such that y = Ax

cy = cAx = Acx (A)

(A) is a vector space in X.

Definition:

The rank of A is defined to be the

dimension of (A).

(i.e.) r(A) = dim (A).

Result 3:

Show that all invertible elements in


n
L(R ) have rank n and conversely.
n
Suppose A L(R ) is invertible.

n n
(i.e.) A: R R is 1 1 and onto.

n
Since A is onto, (A) = R .

n
Therefore r(A) = dim (A) = dim R

= n.

n
Conversely,let the rank of A L(R )

be n.

(i.e.) r(A) = n.

n
(i.e.) dim (A) = dim R

n
Therefore = R .

(i.e.) A is onto.

n
Since R is a finite dimensional vector

space , by theorem 4.1.3,


A is 1 1

Therefore A is invertible.

Projections:

Let X be a vector space. An operator

PL(X) is said to be a projection in X


2
if P = P.

Example: Define P by P(x) = x , xX.

Let x 1 ,x 2 X. Then P(x 1 ) = x 1 , P(x 2 )

= x2.

Now P(x 1 + x 2 ) = P(x 1 )+P(x 2 ) = x 1 +

x 2 and P(cx) = cP(x) = cx.

Therefore P is a linear

transformation.
2
Now P (x) = P(P(x)) = P(x).

Therefore P is a projection.

Result 4:

If P is a projection in X then every

xX has a unique representation of

the form x = x 1 + x 2 where x 1 (P),

x 2 ? (A).

Since x 1 (P) , x 2 ? (A), we have

x 1 = P(x 1 ) and Px 2 = 0.

x 2 = x x 1 P(x 2 ) = P(x) P(x 1 )

0 = x 1 x 1, by putting P(x) = x 1 .

Suppose x = x 3 + x 4 where x 3 (P)

, x 4 ? (A)..

Then x 1 = Px = Px 3 + Px 4 = x 3 + 0 =

x 3 and x 4 = x - x 3 = x - x 1 = x 2
Therefore the expression is unique.

Result 5:

If X is a finite dimensional vector

space and if X 1 is a vector space in X,

then there is a polynomial P in X with

(P) = X 1 .

Since X is finite dimensional and

X 1 X, dim X 1 is finite.

If X 1 contains only 0, this is trivial,

put Px = 0 for all x X 1 .

(P(x 1 +x 2 ) = 0 P(x 1 )+ P(x 2 ) = 0+0

=0 and P(cx) = cP(x) = c0 = 0,

2
P (x) = P(P(x)) = P(0) = 0 = P(x).)

Therefore (P) = X 1 .
Assume dim X 1 = k > 0.

Then it has a basis {u 1 ,u 2 ,...,u k } for

X 1 and {u 1 ,u 2 ,.. .,u n ) for X.

Define P by P(c 1 u 1 +c 2 u 2 +... .+c n u n )

= c 1 u 1 +c 2 u 2 +.. .+c k u k .

Let x X1. Then x =

c 1 u 1 +c 2 u 2 +...+c k u k .

Px = P[c l u 1 +c 2 u 2 +.. .+c k u k ] =

P[c l u l +c 2 u 2 +.. .+c k u k +0.u k+l + ..

.+0.u n ]

=
= c l u l +c 2 u 2 +...+c k u k x.

(i.e.) Px = x , for every x X 1.

Therefore x (P). (i.e.) X 1 (P).

But (P)X 1 .

(P) = X 1 .
(The Rank Theorem)

Theorem 5.2.1:

Suppose m,n,r are non-negative

integers with m r, n r. F is a ?
n
-mapping of an open set E R into
m
R and the derivative F'(x) has rank r

for every xE. Fix aE , put A = F'(a)

, let Y 1 be the range of A, and let


m
P be a projection in R whose range

is Y 1 .Let Y 2 be the null space of P.

Then there are open sets U and V in


n
R ,with aU, U E, and there is a 1

1 ? -mapping H of V onto U (whose

inverse is also of class ? ) such that

F(H(x)) = Ax + (Ax) (xV)

where is a ? -mapping of the open

set A(V) Y 1 into Y 2 .


Proof:

Case (i): Let r = 0. Then rank F(x) =

0 for every xE.

By definition dim {range of F'(x)} =

0.

Range of F'(x) = {0}.

Therefore F'(x) = 0. ||F'(x)|| = 0.

By Mean Value theorem ||F(b)

F(a)|| |b a| ||F'(x)|| = 0

||F(b) F(a)||) = 0.

F(b) = F(a). F(x) is constant.

Let V = U. Define H(x) = x, for every

xV and (0) = F(a).


Now F(H(x)) = F(x) = F(a) = (0) =

0x + (0x).

F(H(x)) = Ax + (Ax) where A = F'(a)

= 0, since F(a) is constant.

Case (ii): Let r > 0. Then rank F'(x)

= r, for every xE.

rank F'(a) = r, for every aE.

rank A = r, since A = F'(a).

dim {range of A} = r.

dim Y 1 = r.

Y 1 has a basis containing r elements,

say {y 1 ,y 2 ,... ,y r }.

n
Choose Z i R so that AZ i = Y i , 1 i

r.
n
Define S: Y1 R by

S(c 1 y 1 +c 2 y 2 +...+c r y r ) =

c 1 z 1 +c 2 z 2 +...+c r z r ,

where c 1 ,c 2 ,...,c r are scalars.

Clearly S is linear.

ASy i = Az i ASy i = y i , 1 i r.

ASy = y, if y Y 1 . (or) AS = I.

n n
Define G: E R R by G(x) = x +

SP[F(x) Ax].

Then G(x+h) G(x) = x +h+

SP[F(x+h) A(x+h)] x SP[F(x)

Ax]

= h+ SP[F(x+h)

F(x) Ax Ah + Ax]
= h+ SP[F(x+h)

F(x) Ah]

G(x+h) G(x) Dh = h+ SP[F(x+h)

F(x) Ah] h[I +SP[F'(x)A],

where

D = I +SP[F'(x)A

SP[F(x+h) F(x) F'(x)+h]

G(x + h) G(x) Dh SP[F(x + h) F(x) F'(x) + h]


h = h

|G(x + h) G(x) Dh| |F(x + h) F(x) F'(x) + h|


|h|
||SP|| |h|

When h 0, the RHS of the

inequality tends to zero.

Therefore LHS tends to zero.


G'(x) = D, for every xE.

(i.e.) G'(x) = = I +SP[F'(x)A].

G'(a) = I +SP[F'(a)A] = I +SP[AA]

( since F'(a) = A)

= I.

n
G'(a) is an identity operator on R .

Clearly G is a ? -mapping on E.

Apply the inverse function theorem to


n
G, there are open sets U and V in R

,with aU,U E.

G(a) V, G is 1 1 and G(U) = V.

G has an inverse H: V U which is

also bijection and H ? (V).


Next to prove that ASPA = A.

n
Let x R . Then ASPA(x) ASP[A(x)]

= ASPy, where y = Ax,

= ASy = Iy = y = Ax.

Therefore ASPA = A.

Since AS = I, PA = A.

Since G(x) = x + SP[F(x) Ax],

AG(x) = Ax + ASP[F(x) Ax] = Ax +

ASPF(x) ASPAx

= Ax + ASPF(x) Ax =

ASPF(x) = PF(x) ( since AS = I)

Therefore AG(x) = PF(x), for every

xE.
In particular, AG(x) = PF(x) holds for

xU. (since U E)

If we replace x by H(x), AG(H(x)) =

PF(H(x)), for every xU=V.

Ax = PF(H(x))

Define (x) = F(H(x)) Ax , for

every x V.

P (x) = PF(H(x)) PAx = Ax Ax =

0.

(x) ? (P) = Y 2 .

Clearly is a ? -mapping of V into

Y2.

To complete the proof, we have to

show that there is a ? -mapping of


A(V) into Y 2 satisfies (Ax) (x), for

every x V.

Put (x) = F(H(x)), for every xV.

'(x) = F'(H(x))H'(x)

Rank of '(x) =

Rank[F'(H(x))H'(x)] = r dim(range of

'(x)) = r.

dim M = r, where M = range of '(x).

Since Y 1 = (A), dim Y 1 = dim

(A)= rank A = r.

Also PF(H(x)) = Ax.

Put (x) Ax P'(x) = A

P maps M into (A) = Y 1 .


P is 1 1 and onto, since M and Y 1

have same dimensions

Suppose Ah = 0, where h = x 2 x 1

P'(x)h = 0 = P.0 '(x)h = 0 .

Again (x) = F(H(x)) Ax, x V

'(x) = F'(H(x))H'(x) A.

'(x) = F'(H(x))H'(x) A = '(x)h

Ah = 0,

Define g(t) = (x 1 +th) where

t[0,1].

g'(t) = '(x 1 +th)h = 0.

Therefore g(t) is constant.


g(0) = g(1).

But g(0) = (x 1 ) and g(1) = (x 2 ).

(x 1 ) = (x 2 ).

Define on A(V) Y 1 such that

(Ax) = (x).

Next to prove that ? (A(V)) ,

Let y 0 be a point in A(V). Then there

exists x 0 V such that y 0 =Ax 0 .

Since V is open and y0 has a

neighborhood W in Y 1 such that

X = X 0 + S(y y 0 ) lies in V,

Ax = Ax 0 + AS(y y 0 ) = y 0 + y y 0

(since AS = I)
= y

(y) = (Ax) = (x) = (x 0 + S(y

y 0 ))

'(y) = (x 0 + S(y y 0 )) S.

? (A(V)).

CYP QUESTIONS:

2 2
x y
1. For (x,y) (0,0), define f= (f1,f2) by f1(x, y) = 2 2
x +y
xy
f2(x, y) = 2 2 compute the rank
x +y

of f(x,y) and find the range of f.

n m
2. Suppose A L(R ,R ), let r be

the rank of A.

a. Define S as in the proof of

theorem 5.2.1. Show that R

(S).
n
is a projection in R whose

null space between and

whose range is (S).

b. Use (a) to show that dim N

(A) + dim R (A)= n.

SECTION-5.3
DETERMINANTS.

Definition:

If (j 1 ,j 2 ,...,j n ) is an ordered n-tuples

of integers.

Define s(j1, j2, ...., jn) = sgn (jq jp)


p<q

where sgn x = 1 if x > 0, sgn x =

1 if x < 0, sgn x = 0 if x = 0. Then

s(j 1 ,j 2 ,..., j n ) = 1, 1 , 0, and it


changes sign if any two of the j's are

interchanged.

Definition:

Let [A] be the matrix of the linear


n
operator A in R with standard basis

{e 1 ,e 2 ,...,en }. Let a(i,j) be the entry

in the ith row and jth column of [A].

We define

det [A] =

s(j 1 ,j 2 ,...,j n )a(1,j 1 )a(2,j 2 )....a(n,j n ) .

the sum extends over all ordered n-

tuples of integers (j 1 ,j 2 ,...,j n ) with

1 jn.
Example:

[ ]
2 4
If [A] = , then det[A] = s(j1, j2)a(1, j1)a(2, j2)
5 3

=
s(1,2)a(1,1)a(2,2) +

s(2,1)a(1,2)a(2,1)

= 1.2.3 +
(1)4.5 = 6 20 = 14

Note : Let [A] be the matrix.

[ ]
a11 a12 a1j a1n

a21 a22 a2j a2n


[A] =

an1 an2 anj ann


n n

The jth column vector xj = a1je1 + a2je1 + .... + anjen = aijei = a(i, j)e i
j=1 i=1

Therefore det[A] = det (x 1 ,x 2 ,..

.,x n ).

Theorem 5.3.1:
n
a. If I is the identity operator on R

then det[A] = det (x 1 ,x 2 ,.. .,x n )

= 1.

b. det is a linear function of each

of the column vectors x, if the

others are held fixed.

c. If [A] 1 is obtained from [A] by

interchanging two columns, then

det[A] 1 = det[A].

d. If [A] has two equal columns,

then det[A] = 0.

Double click this page to view clearly


Proof:

a. If the matrix [A] = [I] then a(i,j)

{
1 if i = j
a(i, j) =
0 if i i

det[I] = s(j 1 , j 2 ,... j n )a(1,

j 1 )a(2,j 2 )....a(n,j n )

= s( 1,2,... ,n)a( 1,1

)a(2,2).. ..a(n,n)

= s(1,2,...,n).1

= sgn(2 1)sgn(3

2).....sgn(n (n 1))

= 1.

b. Since s(j 1 ,j 2 ,...,j n ) =

sgn (jq jp) where sgn(j q


p<q

j p ), where sgn x = 1 if x > 0,

sgn x = 1 if x < 0, sgn x =


=
0 if x = 0, s(j 1 ,j 2 ,.. .,j n ) 0 if
any two of the j's are equal. Each

of the remaining n! products in

det [A] =
s(j 1 ,j 2 ,...j n )a(1,j 1 )a(2,j 2 )....a(n,j n )

contains exactly one factor from

each column. Therefore det is a

linear function of each of the

column vectors x, if the others

are held fixed.

c. In s(j 1 ,j 2 ,...,j n ) if two entries are

interchanged, it effects the

change of sign. Therefore


det[A] 1 = det[A].

d. Suppose two columns of [A] are

equal, then interchange the two

columns, we get det[A] 1 =

det[A].
But [A] = [A] 1 .

Therefore det[A] = det[A].

(i.e.) 2det[A] = 0. (i.e.) det[A] = 0.

Theorem 5.3.2:

If [A] and [B] are n by n matrices

then

det([B][A]) = det[B]det[A].

Proof:

Let x 1 ,x 2 ,.. .,x n be the columns of

[A].

Define B(x1, x2, ....,xn) = B[A] = det([B][A]) ......(1)

(i.e.) Bx 1 ,Bx 2 ,...,Bx n are the column

vector of [B][A].

(i. e.) B(x1, x2, ...., xn) = det (Bx1, Bx2, ....., Bxn). .....(2)
By (2) and theorem 5.3.1, b also

has the properties (b) to (d).

[ ]
n n

Since xj = a(i, j)e , [A] = a(i, j)e , x , ..., x


i B B i 2 n
i=1 i=1

= a(i, 1) [e , x , ..., x ].
B i 2 n
i=1

Repeating this process with x 2 ,... ,x n

we obtain

B[A] = a(i , 1)a(i , 2)..........a(i , n) (e , e , .....e )


1 2 n B i1 i2 in

where the sum being extended over

all ordered n-tuples (i 1 ,i 2 , .... i n ) with

the condition 1 i r n .

Now B [e i ,x 2 ,...,x n ] = t(i 1 ,i 2 ,.... i n ).

B (e 1 ,e 2 ,...,e n )

Double click this page to view clearly


Since [B][I] = [B], det([B][I]) =

det[B].

B (e 1 ,e 2 ,...,e n ) = B [I] =

det([B][I])

b [A] =
a(i 1 ,1)a(i 2 ,2)....a(i n ,n)t(i 1 ,i 2 ,....

i n ). B (e 1 ,e 2 ,...,e n ) det([B][A]) =

B [A] = a(i 1 ,1)a(i 2 ,2)....a(i n ,n)

t(i 1 ,i 2 ,.... i n ).det[B]- (1)

Take [B] = [I], we get

det[A] = a(i 1 ,1)a(i 2 ,2)....a(i n ,n)

t(i 1 ,i 2 ,.... i n ).det[I]

= a(i 1 , 1 )a(i 2 ,2)....a(i n ,n)

t(i 1 ,i 2 ,.... i n ) ( since det[I] = 1)

Substitute in (1), we get


det([B][A]) =

det[B]det[A].

Theorem 5.3.3:

A linear operator A on is invertible iff

det[A] 0.

Proof:

Suppose A is invertible.

1
Then [A][A ] = I.

1
By theorem 5.3.2, det[A]det[A ] =
1
det([A][A ]) = det[I] = 1.

Therefore det[A] 0.

Conversely, let det[A] 0.

Now to prove that A is invertible.


Suppose A is not invertible.

Then the column vectors x 1 ,x 2 ,... .,x n

of [A] are dependent.

Therefore there exists a vector x k

such that xk + c x =0 j j for some


jk

scalars c j . If we replace x k by x k

+ c j x j , then the determinant of the

matrix is unchanged. The same result

is true if we replace x k by

xk + c x j j (i.e. zero). But the


jk

determinant of a matrix contains a

column of zeros is zero.

(
det[A] = det x1, x2, ....., xk + c x , ....x
jk
j j
)
n = det(x1, x2, ...., 0, ...., xn)

Double click this page to view clearly


 ZKLFK LV D FRQWUDGLFWLRQ

WRRXUDVVXPSWLRQ

L$LVLQYHUWLEOH

)+7*1)6;

Q P
,I I PDSV DQ RSHQ VHW (O 5 LQWR 5 

DQG LI I LV GLIIHUHQWLDEOH DW D SRLQW [

F ( WKH GHWHUPLQDQW RI WKH OLQHDU

RSHUDWRU I
[ LV FDOOHG WKH -DFRELDQ

RI I DW [ ,Q V\PEROV - I [ GHW I [ 
\  \Q
:H DOVR XVH WKH QRWDWLRQ IRU
[  [Q

- I [ LI \  \  \ Q I [  [  [ Q

  
: 
" "

-.161<176
6XSSRVH I LV D UHDO IXQFWLRQ GHILQHG
Q
LQ DQ RSHQ VHW (O 5 ZLWK SDUWLDO
derivatives D 1 f, D 2 f,..., D n f . If the

function Djf are themselves

differentiable, then the second order

partial derivative of f are defined by

D ij f= D i D j f where i = l,2,...,n, j =

l,2,...,n. If all these functions D ij f are

continuous in E then we say that f is

of class ? in E (or) f ? (E)

Note: If the derivatives are


continuous then D ij f = D ji f.

Theorem 5.4.1:

Suppose f is defined- in an open set


2
E R , and D i f, D 21 f are exist at

every point of E. Suppose Q E is a

closed rectangle with sides parallel to


the coordinate axes, having (a,b) and

(a+h,b+k) as opposite vertices (h

0, k 0). Put

(f,Q) = f(a+h,b+k) f(a+h,b)

f(a,b+k) + f(a,b).

Then there is a point (x,y) in the

interior of Q such that

(f,Q) = hk(D 21 f)(x,y).

Proof:

Let u(t) = f(t, b + k) f(t, b). .....(1)

Given (f,Q) = f(a+h,b+k)

f(a+h,b) f(a,b+k) + f(a,b).

= u(a+h) u(a) = h

u'(x) where x is a point lies between

a and a+h.
Differentiate (1) with respect to t, we

get

u'(t) = D 1 f(t,b+k) D 1 f(t,b)

u'(x) = D 1 f(x,b+k) D 1 f(x,b)

(f,Q) = h u'(x) = h[D 1 f(x,b+k)

D 1 f(x,b)]

= hk(D 21 f)(x,y) where b <

y < b+k.

Theorem 5.4.2:

Suppose f is defined in an open set


2
E R , and suppose D 1 f, D 21 f and

D 2 f are exist at every point of E and

D 21 f. is continuous at some point

(a,b) E. Then D 21 f exists at (a,b)

and
(D 12 f)(a,b) = (D 21 f)(a,b).

Proof:

Suppose A = (D 21 f)(a,b).

Given that (D 21 f.) is continuous at

(a,b) E.

Let > 0 be given.

If Q is a rectangle contained in E such

that |(D 21 f)(x,y) (D 21 f)(a,b)|<

(i.e.) |(D 21 f)(x,y) A | < .

By theorem 5.4.1, (f,Q) =

hk(D 21 f)(x,y).

(f, Q)
hk = (D21f)(x, y).

| |
(f, Q)
hk A <
(f, Q)
lim hk = A .....(1)
h 0, k 0

Since (f, Q) = f(a + h, b + k) f(a + h, b) f(a, b + k) + f(a, b),

(f, Q) f(a + h, b + k) f(a + h, b) f(a, b + k) + f(a, b)


hk = hk

=
1
hk [f(a + h, b + k) f(a + h, b) (f(a, b + k) f(a, b))]

[( ]
1 f a + h, b + k) f(a + h, b) f(a, b + k) f(a, b)
= h k k

Keeping h fixed and allowing k 0,

(f, Q)
lim hk
k0

=
1
h
{[ lim
k0
f(a + h, b + k) f(a + h, b)
k
] [

k0
lim
f(a, b + k) f(a, b)
k
]}
=
1
h [(D2f)(a + h, b) (D2f)(a, b)]
[(D2f)(a + h, b) (D2f)(a, b)] = D12f(a, b)
(f, Q) 1
lim hk = lim h
h 0, k 0 h0

From A = D 12 f(a,b)

Therefore (D 21 f)(a,b) = (D 12 f)(a,b).

CYP QUESTIONS:

1. Show that the existence of D 12 f

does not imply the existence of

D 21 f. For example, let f(x,y) =

g(x), where g is nowhere

differentiable.

Double click this page to view clearly


SECTION-5.5
DIFFERENTIATION OF INTEGRALS

Theorem 5.5.1: Suppose


a. (x,t) is defined for a x b, c

t d;

b. is an increasing function

on[a,b];

c. 'R () for every t [c,d];

d. c < s < d, and to every > 0


corresponds a > 0 such that

|(D 2 (x,t) (D 2 )(x,s)| < for

all x [a,b] and for all t (s ,

s +).

Define f(t) = (x, t) d(x) ( c t d)


a

s
Then (D 2 ) (),f'(s) exists,
b

and f' (s) = (D )(x, s) d(x)


2
a

Proof:

Consider the difference quotients

(x, t) (x, s)
(x, t) = ts for 0 < | t s | < .

By Mean vale theorem,

(x,t) (x,s) = (t s) D 2 (x,u),

where t < u < s.

(x, t) (x, s)
ts = D2(x, u).

(i.e.) (x,t) = D 2 (x,u).

By (d), |(D 2 (x,t) (D 2 (x,s)| <

for all x [a,b] and for all t (s ,

s +).

(i. e.) |(x, t) D2(x, s)| < .....(1)


b

Define f(t) = (x, t) d (x).


a

Then f(s) = (x, s) d (x).


a

b
(x, t) (x, s)

f(t) f(s)
= d (x)
ts ts
a

= (x, t) d(x)
a

From (1), (x,t) converges to

D 2 (x,s),

t s
Therefore (x) (D 2 ) uniformly

on [a,b].

By theorem 2.4.1, is monotonically

increasing on [a,b] and


f n ().on [a,b], for n = 1,2,3,....,

and suppose f n f uniformly on

[a,b]. Then f ()on [a,b], and


b b

f d = n
lim
a
f n d
a

s t
we have (D 2 ) () (since

())

(x, t) d (x),
f(t) f(s)
Since ts =
a

b b

(x, t) d(x) =
f(t) f(s)
lim ts = lim lim (x, t) d (x)
ts ts ts
a a

= (D )(x, s) d(x) (by (1))


2
a

herefore f'(s) = (D )(x, s) d (x)


2
a

Double click this page to view clearly


Example:

Compute the integrals


e
2
x
f(t) = cos(xt) dx cos(xt)dx

and


2
x
g ( t) = xe sin (xt) dx, where < t < .

We claim that f is differentiable and

f(t) = g(t).
cos( + ) cos
If > 0, then + sin
sin sin + cos ( +) cos
= + sin

sin sin + cos( + ) cos + sin


=

( + ) sin + cos( + ) ( sin + cos )


=

=
1
( sin sin t ) dt

| |
+

| | ( sin sin t ) dt
cos( + ) cos 1
+ sin =

Double click this page to view clearly


+

||
1
| sin sin t | dt

+

||
1
| t | |dt|

(by Mean value theorem)

[ ]
+
[ ]
2
1 ( t ) 1 2
|| 2 || ( + ) 0

| 2| =
1 2 1 ||
|| = || 2

| |
cos( + ) cos ||
(i. e.) + sin 2 .....(1)


2
x
Given that f(t) = e cos(xt) dx

e e
2 2
cos(x(t + h)) dx
x x
f(t + h) f(t) = cos(xt) dx

e [cos(xt + xh)) cos (xt)] dx


f(t + h) f(t) 1 x
2

h = h

Let = xt, = xh.


f(t + h) f(t)
h g ( t)

[ ] xe
2 2
cos(xt + xh)) cos(xt) dx +
1 x x
= h e sin(xt) dx

e [cos(xt + xh)) cos(xt) + xh sin(xt)] dx


2
1 x
= h

e
2

[cos( + ) cos + sin ] dx


1 x
= h

| | e
f(t + h) f(t) 2

|cos( + ) cos + sin | dx


1 x
h g(t) |h|

| | ||
cos( + ) cos + sin
From (1) 2

2
|| 2
(i. e.) |cos( + ) cos + sin | 2 < | | .

Double click this page to view clearly


| | e
f(t + h) f(t) 1
2
x 2
h g(t) < |h| || dx

e |x| |h| dx = e |x| dx = e


2 2 2
1 x 2 2 x 2 x 2
< |h|
x dx

When h 0, f is differentiable at t

and f(t) = g(t).

Next to find f(t):


2
x
f(t) = e cos(xt)dx.

2
x
Let u = e , dv = cos(xt) dx.
2
v = sin(xt) / t
du x
Then dx = 2xe

[ ]
2
x
sin xt

e sin(xt) x
2
f ( t) = +2 xe dx
t

t


2
2 x 2
=0+ t xe sin xtdx = t g(t)

(i. e.) tf(t) = 2g(t)

Since f(t) = g(t), f satisfies the

differential equation 2f(t) + tf(t) = 0

Double click this page to view clearly


I. F = e t
2
dt
=e
2
t
4


2
t
Therefore e 4 f(t) = 0 + c= c.

e e
2 2
x x
Initially, when t = 0, f(0) = dx + 2 dx
0
2 dz
Let z = x . Then dx = 2x.
dz dz
(i. e.) dx = 2x = d(z)

1

1 1
z z 1 z
f(0) = 2 e dz = z 2
e dz = z2 e dz =
2z
0 0 0

c = .

2
t
The required solution is e f(t) = . 4

2
t

f(t) = e 4
.

CYP QUESTIONS:

1. For t 0, put

{
x ( 0 x t)
(x, t) = x + 2t (t x 2t)
0 otherwise

and put (x,t) = (x,|t|) if t <

0.

Double click this page to view clearly


2
Show that is continuous on R ,

and (D 2 )(x,0) = 0 for all x.


1

Define f(t) = (x, t) dx


1
1
Show that f(t) = t if |t| < 4 Hence
1

f'(0) (D )(x, 0) dx
2
1

Vous aimerez peut-être aussi